+7 495 120-13-73 | 8 800 500-97-74

(для регионов бесплатно)

Содержание

формула для расчета электрической емкости

Конденсатор – радиоэлектронный прибор, способный накапливать и отдавать заряд. Как правило, на его корпусе дается информация о его емкости, но иногда требуется самому рассчитать этот номинал. Конденсаторами могут выступать и проводники, они также обладают определенной емкостью. Для расчета существует несколько формул емкости конденсатора, их и рассмотрим.

В чем измеряется емкость конденсатора

Что такое заряд еще проходят в школе, когда эбонитовую палочку натирают о шерстяную ткань и подносят к маленьким кусочкам бумаги. Под действием электромагнитных сил бумага прилипает к палочке. Подобный заряд накапливается в конденсаторе. Но для начала познакомимся с самим конденсатором.

Простейшим конденсатором являются две металлические пластины, разделенные диэлектриком. От качества диэлектрика зависит, как долго энергия заряженного конденсатора может сохраняться. На этих пластинах, они еще называются обкладками, накапливается разноименный заряд.

Как это происходит?

Электрический заряд, а в случае с металлами это электроны, способен перемещаться под действием электродвижущей силы (э. д. с.). Подключая металлические пластинки к источнику тока, мы получаем замкнутую цепь, но разделенную диэлектриком. Электростатическое поле проходит этот диэлектрик, замыкая цепь, а электроны, дойдя до препятствия, останавливаются и скапливаются.

Получается, на одной обкладке наблюдается избыток электронов, и эта пластина имеет отрицательный знак, а на другой пластине электронов недостает настолько же, знак на этой обкладке, конечно же, будет положительным.

Вот теперь нужна для определения емкости конденсатора

формула, определяющая, какой заряд способен разместится на конкретном конденсаторе.

В качестве единицы измерения в международной системе (СИ) емкость определяется в Фарадах.

Много это или мало — емкость в 1Ф? Чтобы конденсатор обладал емкостью в 1Ф, он должен содержать в себе заряд в 1К (кулон) и при этом напряжение между обкладками должно равняться 1 вольту.

Интересно. Что такое заряд в 1 кулон? Если два предмета, каждый из которых имеет заряд в один кулон разместить в вакууме на расстоянии один метр, то сила притяжения между ними будет равна силе притяжения землей тела массой в один миллион тонн.

Как и любая буквальная емкость один и тот же конденсатор может вмещать разное количество заряда.

Рассмотрим пример.

  • В трехлитровую банку входит три литра воздуха. Его хватит для дыхания, допустим, на 3 минуты. Но если воздух закачать под каким-то давлением, то емкость так и останется три литра, однако дышать можно будет дольше. Так устроен акваланг для ныряльщиков. Получается, количество воздуха в банке зависит от давления, которое в ней создается. Точно так же есть некая зависимость между различными силами, влияющими на емкость.

Формула емкости плоского конденсатора

Прежде чем узнать, по какой формуле вычисляется емкость плоского конденсатора, рассмотрим формулу для одиночного проводника. Она имеет вид:

  • где Q – заряд,
  • φ – потенциал.

Как видно емкость конденсатора, формула которого здесь приведена, будет тем больше, чем больший заряд способен накапливаться на нем при незначительном потенциале. Чтобы легче это было понять, рассмотрим получившие широкое распространение плоские конденсаторы разных размеров.

Для получения качественного конденсатора важны любые мелочи:

  1. ровная поверхность каждой обкладки;
  2. обе пластинки по всей площади должны располагаться на одинаковом расстоянии;
  3. размеры обкладок должны быть строго идентичными;
  4. от качества диэлектрика, расположенного между пластинками, будет зависеть ток утечки;
  5. емкость напрямую зависит от расстояния между обкладками, чем оно меньше, тем больше емкость.

Теперь обратимся к плоскому конденсатору. Формула определения емкости конденсатора несколько отличается от приведенной выше:

  • где S – площадь одной обкладки,
  • ε— диэлектрическая проницаемость диэлектрика,
  • ε0 — электрическая постоянная,
  • d – расстояние между обкладками.

Электрическая постоянная выражается числом 8,854187817×10-12.

Внимание! Эта формула справедлива только тогда, когда расстояние между пластинами намного меньше их площади.

Попробуем разобраться с каждой переменной подробнее. Площадь измеряется в м2, точнее, приводится к этой величине. А вот проницаемость диэлектрика может обозначаться по-разному.

В России это ε(также означает относительная проницаемость), в англоязычной литературе встречается ε(также означает абсолютная проницаемость), а то может и вовсе использоваться без индекса, просто ε. О том, что здесь используется диэлектрическая проницаемость диэлектрика можно понять из контекста.

Дальше идет ε0. Это уже вычисленное значение, измеряемое в Ф/м. Последняя переменная – d. Измеренное расстояние также приводится к метру. Емкость конденсатора, формула которого сейчас рассматривается, показывает сильную зависимость от расстояния обкладок. Поэтому стараются это расстояние по возможности сокращать. Почему этот показатель так важен?

Идеальными условиями для получения наибольшей емкости – это отсутствие промежутка между обкладками, чего, конечно, добиться невозможно. Чем ближе находятся разноименные заряды, тем сильнее сила притяжения, но здесь возникает компромисс.

При уменьшении толщины диэлектрика, а именно он разделяет разноименные заряды, возникает вероятность его пробоя из-за разности потенциалов на обкладках. С другой стороны, как уже говорилось, при увеличении напряжения увеличивается количество зарядов. Вот и приходится выбирать между емкостью и рабочим напряжением конденсатора.

Есть другая формула для плоского переменного конденсатора:

Здесь диэлектрическая проницаемость обозначена буквой ε, π = 22/7 ≈ 3,142857142857143, d – толщина диэлектрика. Формула предназначена для конденсатора, состоящего из нескольких пластин.

Допустимая толщина диэлектрика d также зависит от εr, чем выше коэффициент, тем тоньше можно использовать диэлектрик, тем большую емкость будет иметь конденсатор. Это был самый сложный материал, дальше будет легче.

Формула емкости цилиндрического конденсатора

Теперь поговорим о том, как найти емкость конденсатора цилиндрической формы. К ним относятся конденсаторы, состоящие из двух металлических цилиндров, вставленных один в другой. Для разделения между ними расположен диэлектрик. Формула емкости конденсатора выглядит следующим образом:

Здесь видим несколько новых переменных:

  • l – высота цилиндра;
  • R1 и R2 – радиус первого и второго (внешнего) цилиндров;
  • ln – это не переменная, а математический символ натурального логарифма. На некоторых калькуляторах он имеется.

Всегда нужно помнить, что все величины должны приводиться к единой системе, в приведенной ниже таблице указаны международные системы единиц (СИ).

Из нее видно, что все расстояния нужно приводить к метру.

Еще стоит обращать внимание на качество диэлектрика. Если толщина диэлектрика влияет только на емкость конденсатора, то его качество затрагивает сохранность энергии. Другими словами, конденсатор с качественным диэлектриком будет иметь меньший саморазряд.

Определить качество можно по числу, стоящему возле вещества, чем оно больше, тем лучше качество. Сравнение производится по вакууму, значение которого равно единице.

Формула емкости сферического конденсатора

Последнее что осталось разобрать – формулу определения емкости конденсатора, состоящего из двух сфер. Причем одна сфера находится внутри другой. Формула имеет следующий вид:

Из приведенных переменных здесь все знакомо. Стоит обратить внимание лишь на сам конденсатор.

Кроме своей необычной формы у него есть свои особенности: внутри малой сферы никакого заряда нет, он образуется на внешней части малой сферы и внутренней части большого шара. Также заряд отсутствует и на внешней стороне внешней сферы.

Так же как и все другие конденсаторы, сферы разделены диэлектриком. Толщина и качество диэлектрика оказывают такое же влияние на емкость, как в случае с другими конденсаторами.

После того как были рассмотрены формулы, стоит испробовать их на практике. Рассмотрим, как найти емкость конденсатора каждого вида.

Примеры решения задач

Начнем с плоского конденсатора. Формула для этого вида:

Допустим, у нас есть следующие значения:

  • в качестве диэлектрика возьмем слюду толщиной 0,02 мм, ε = 6;
  • конденсатор квадратный со сторонами в 7 мм.

Определяем площадь пластин: 7×7 = 49 мм2.

Приводим к единой системе: 4,9×10-5 = 0,000049 м2. Толщина диэлектрика 0,02×10-5 = 0,00002 м. Электрическая постоянная 8,854187817×10-12.

Подставляем в формулу и высчитываем числитель: 6×8,854187817×10-12 ×4,9×10-5, сокращаем и решаем 6×49×8,854187817×10-17 = 2,603131218198×10-14.

Делим на толщину диэлектрика: 2,603131218198×10 / 2×10 = 1301,565609099×10 = 1,301565609099×10. Шесть нулей – это тысячи или приставка «микро», получается округлено 1,3 мкФ.

Возможно, при вычислении была допущена ошибка, но это не экзамен по математике. Важно понять сам метод вычисления.

Формула для цилиндрического конденсатора:

Выбираем значения:

  • l = 1 см;
  • R1 = 0,25 мм;
  • R2 = 0,26 мм;
  • ε = 2.

Подгоняем под единую систему: l — 1 см = 1×10-2 = 0,01 м; R1 – 0,25 мм = 0,0025 м; R2 – 0,26 мм = 0,0026 м.

Подставляем значения в числитель: 2×3,142857142857143×8,854187817×10-12×2×0,01 1,11×10-12. Находим знаменатель: 0,26:0,25 = 1,04.

Находим натуральный логарифм, он равен примерно 0,39. Числитель делим на знаменатель: 1,11×10-12/0,39 = 2,85×10-12.

Число с 12 нулями это приставка «пико», получаем 2,85 пФ.

Формула для сферического конденсатора:

Выбираем значения:

  • ε= 4;
  • r1= 5 см;
  • r2= 5,01 см.

Снова все подгоняем: 5 см = 0,05 м; 5,01 см = 0,0501 м. Заполняем числитель. 4×3,142857142857143×4×8,854187817×10-12×0,05×0,0501 1,11×10-12 Вычисляем знаменатель: 0,0501 – 0,05 = 0,01. Производим деление: 1,11×10-12×0,01 = 1,11×10-10. Снова получили пикофарады, а именно 1,11 пФ.

Похожие материалы на сайте:

Понравилась статья — поделись с друзьями!

 

По какой формуле найти ёмкость (объем) конденсаторов

Во всех электронных устройствах используются конденсаторы. При их конструировании или изготовлении своими руками параметры устройств рассчитываются по специальным формулам.

Конденсаторы

Расчёт конденсаторов

Один из главных параметров таких устройств – ёмкость. Рассчитать её можно по следующей формуле:

C=q/U, где:

  • C – ёмкость,
  • q – заряд одной из обкладок элемента,
  • U – разность потенциалов между обкладками.

В электротехнике вместо понятия «разность потенциалов между обкладками» используется «напряжение на конденсаторе».

Ёмкость элемента не зависит от конструкции и размеров устройства, а только от напряжения на нём и заряда обкладок. Но эти параметры могут изменяться в зависимости от расстояния между ними и материала диэлектрика. Это учитывается в формуле:

С=Co*ε, где:

  • С – реальная ёмкость,
  • Со – идеальная, при условии, что между пластинами вакуум или воздух,
  • ε – диэлектрическая проницаемость материала между ними.

Например, если в качестве диэлектрика используется слюда, «ε» которой 6, то ёмкость такого устройства в 6 раз больше, чем воздушного, а при изменении количества диэлектрика меняются параметры конструкции. На этом принципе основана работа ёмкостного датчика положения.

Устройство конденсатора

Единицей ёмкости в системе СИ является 1 фарад (F). Это большая величина, поэтому чаще применяются микрофарады (1000000mkF=1F) и пикофарады (1000000pF=1mkF).

Расчет плоской конструкции

Если нужно рассчитать плоский конденсатор, то необходимо учесть площадь обкладок и расстояние между ними. Это отражено в формуле, по которой рассчитывается ёмкость плоского конденсатора:

C=ε/d, где:

  • ε – диэлектрическая проницаемость изолирующего материала,
  • d – расстояние между пластинами.

Расчет конструкции цилиндрической формы

Цилиндрический конденсатор – это две соосные трубки различного диаметра, вставленные друг в друга. Между ними находится диэлектрик. При радиусе цилиндров, близком друг к другу и намного большем, чем расстояние между ними, цилиндрической формой можно пренебречь и свести расчёт к формуле, аналогичной той, по которой рассчитывается плоский конденсатор.

Вычисляются параметры такого устройства по формуле:

C=(2π*l*R*ε)/d, где:

  • l – длина устройства,
  • R – радиус цилиндра,
  • ε – диэлектрическая проницаемость изолятора,
  • d – его толщина.

Расчёт сферической конструкции

Есть устройства, обкладки которых представляют собой два шара, вложенные друг в друга. Формула ёмкости такого прибора:

C=(4π*l*R1*R2*ε)/(R2-R1), где:

  • R1 – радиус внутренней сферы,
  • R2 – радиус внешней сферы,
  • ε – диэлектрическая проницаемость.

Формулы ёмкости конденсаторов различной формы

Ёмкость одиночного проводника

Кроме конденсаторов, способностью накапливать заряд обладают отдельные проводники. Одиночным проводником считается такой проводник, который бесконечно далёк от других проводников. Параметры заряженного элемента рассчитывается по формуле:

C=Q/φ, где:

  • Q – заряд,
  • φ – потенциал проводника.

Объём заряда определяется размером и формой устройства, а также окружающей средой. Материал прибора значения не имеет.

Способы соединения элементов

Не всегда есть в наличии элементы с необходимыми параметрами. Приходится соединять их различными способами.

Соединение конденсаторов

Параллельное соединение

Это такое соединение деталей, при котором к одной клемме или контакту присоединяются первые обкладки каждого конденсатора. При этом вторые обкладки присоединяются к другой клемме.

При таком соединении напряжение на контактах всех элементов будет одинаковым. Заряд каждого из них происходит независимо от остальных, поэтому общая ёмкость равна сумме всех величин. Её находят по формуле:

C=C1+C2+…Cn,

где C1-Cn – параметры деталей, участвующих в параллельном соединении.

Важно! Конденсаторы имеют предельное допустимое напряжение, превышение которого приведёт к выходу элемента из строя. При параллельном соединении устройств с различным допустимым напряжением этот параметр получившейся сборки равен элементу с наименьшим значением.

Последовательное соединение

Это такое соединение, при котором к клемме присоединяется только одна пластина первого элемента. Вторая пластина присоединяется к первой пластине второго элемента, вторая пластина второго – к первой пластине третьего и так далее. Ко второй клемме присоединяется только вторая обкладка последнего элемента.

При таком соединении заряд на обкладках конденсатора в каждом приборе будет равен остальным, однако напряжение на них будет разным: для зарядки устройств большей ёмкости тем же зарядом требуется меньшая разность потенциалов. Поэтому вся цепочка представляет собой одну конструкцию, разность потенциалов которой равна сумме напряжений на всех элементах, а заряд конденсатора равен сумме зарядов.

Последовательное соединение увеличивает допустимое напряжение и уменьшает общую ёмкость, которая меньше самого меньшего элемента.

Рассчитываются эти параметры следующим образом:

  • Допустимое напряжение:

Uобщ=U1+U2+U3+…Un, где U1-Un – напряжение на конденсаторе;

  • Общая ёмкость:

1/Собщ=1/С1+1/С2+1/С3+…1/Сn, где С1-Сn – параметры каждого устройства.

Интересно. Если в цепи только два элемента, то можно воспользоваться упрощённой формулой: Собщ=(С1*С2)/(С1+С2).

Смешанное соединение

Это такое соединение, в котором есть детали, соединённые последовательно, и есть соединённые параллельно. Параметры всей цепи рассчитывается в следующей последовательности:

  1. определяются группы элементов, соединённые параллельно;
  2. для каждой группы в отдельности рассчитывается эквивалентные значения;
  3. рядом с каждой группой параллельно соединённых деталей пишутся получившиеся величины;
  4. получившаяся схема эквивалентна последовательной схеме и рассчитывается по соответствующим формулам.

Знание формул, по которым можно найти емкость при изготовлении конденсаторов или их соединении необходимо при конструировании электронных схем.

Видео

Оцените статью:

проверенный способ соединения, формула, типы подключений

Если нужно срочно отремонтировать технику, а нужного конденсатора нет, то можно увеличить емкость конденсатора, как известно из школьной программы, соединив несколько приборов в одну цепь.

Такая проблема может также возникнуть, если, например, нужного номинала нет в продаже, то есть для нестандартных подключений, например, в радиотехнических опытах.

Электрическая емкость

При соединении приборов для конденсации заряда, как правило, техника интересует электрическая емкость, которая получится в итоге.

Электроемкость показывает способность двухполюсника накапливать в себе заряд и измеряется в фарадах. Может показаться, что чем выше это значение, тем лучше, но на практике не существует возможности создать все возможные на свете емкости, более того, часто это и не нужно, так как во всех приборах, использующихся повседневно, применяются стандартные приборы для конденсации.

Можно соединить несколько приборов для конденсации в цепь, создав одну конденсирующую емкость, при этом значение характерной величины будет зависеть от типа подключения, и для его расчета есть давно известные формулы.

Параллельное соединение

Существует два типа подключения приборов в цепь: последовательное и параллельное. Каждый из них обладает своими свойствами, но, как правило, используется параллельное соединение конденсаторов.

Параллельное соединение обладает такими свойствами:

  1. Емкость составного двухполюсника увеличивается по сравнению с каждым отдельным прибором.
  2. Напряжение в сети не изменяется.

Соединить конденсаторы для увеличения емкости, как показывают свойства, лучше этим способом. Для этого нужно соединить выводы с каждого двухполюсника по группам: у каждого из них два вывода. Нужно создать две группы: в одну соединить все конденсаторы с одного вывода, а во вторую с оставшегося.

При таком соединении приборы для конденсации образуют одну емкость, поэтому верна такая формула: С=С1+С2+…СN, где N — количество конденсаторов в цепи.

Например, если имеются номинальные значения 50мкф, 100мкф и 150мкф, то при последовательном подключении общее значение в цепи будет 300мкф.

В жизни это подключение используют довольно часто, например, если при расчетах оказалось, что требуется такой двухполюсник, которого в продаже точно не найти. С помощью этого способа можно варьировать емкость конденсатора так, как это потребуется, при этом не изменяя напряжение в сети.

Последовательное включение конденсаторов

Свойства последовательного включения конденсаторов:

  1. Емкость последовательно соединенных приборов для конденсации заряда в отличие от емкости параллельно соединенных конденсаторов уменьшается.
  2. Напряжение на приборах растет.

Для такого подключения нужно просто соединять выводы двухполюсников один с другим, образуя цепочку: вывод первого будет соединен с выводом второго, оставшийся вывод второго с выводом третьего и так далее.

Формула подключения: 1/(1/С1+1/С2+…+1/СN), где N — это количество приборов в соединении.

Например, есть три конденсатора по 100мкф. 1/100+1/100+1/100=0,03мкф. 1/0,03=33мкф.

Заряды распределятся с чередующимся знаком, а емкостное значение будет ограничено только им же для самого слабого звена в цепи. Как только он получит свой заряд, передача тока в цепи прекратится.

Для чего тогда нужен подобный способ подключения? Такая цепь более устойчива и может выдержать большее напряжение при подключении в схему при меньшем емкостном номинале конденсатора. Однако в продаже имеются приборы, которые и без того обладают нужными свойствами, поэтому-то такое подключение в жизни практически не используется, а если используется, то для специфических задач.

Смешанный способ

Сочетает в себе параллельное и последовательное подключения.

При этом для участков с последовательным соединением характерны свойства последовательного соединения, а для участков с параллельным — свойства параллельного.

Оно используется, когда ни электроемкость, ни номинальное напряжение приборов, имеющихся в продаже, не подходят для задачи. Обычно такая проблема возникает в радиотехнике.

Чтобы определить общее значение электроемкости, нужно будет сначала определить это же значение для параллельно соединенных двухполюсников, а потом для их последовательного соединения.

Сравнение различных вариантов

 ЕмкостьНапряжение
ПараллельноеУвеличиваетсяНе изменяется
ПоследовательноеУменьшаетсяУвеличивается
СмешанноеИзменяетсяУвеличивается

Для выбора соединения можно воспользоваться такой таблицей. Слева тип соединения приборов, сверху свойства прибора для конденсации заряда.

Если требуется увеличить емкость, то нужно использовать параллельное соединение, а если увеличить напряжение — то последовательное. Если же требуется и то, и то, то нужно будет рассчитывать смешанное подключение конденсаторов в цепь.

Плавное регулирование емкости конденсаторов

Введение

Регулирование емкости конденсаторов требуется как в электротехнических, так и в радиотехнических устройствах. В радиотехнических устройствах, например в параметрических усилителях, регулирование емкости конденсатора, представляющего собой полупроводниковый переход, смещенный в обратном направлении, осуществляется регулированием напряжения смещения [1]. Небольшая емкость перехода на радиочастотах оказывается вполне достаточной.

В электротехнике регулирование емкости возбуждающих конденсаторов асинхронного генератора используется, например, для регулирования его выходного напряжения [2]. Поскольку частоты генерируемых напряжений на много порядков ниже, чем в радиотехнике, емкость полупроводникового перехода оказывается ничтожно малой по сравнению с необходимой емкостью. Поэтому используются нелинейные конденсаторы (вариконды) [2]. Их емкость изменяется в зависимости от приложенного напряжения благодаря особым свойствам диэлектрика.

В электротехнике широко применяются импульсные методы регулирования не только величин токов и напряжения, но даже их частоты. Последнее время благодаря успехам в силовой электронике и микропроцессорной технике импульсные методы регулирования применяются уже в устройствах, мощность которых достигает десятков мегаватт. Это наводит на мысль об импульсном регулировании параметров элементов электротехнических устройств, например емкости конденсаторов.

 

Идея импульсного регулирования емкости конденсатора

Рассмотрим конденсатор постоянной емкости, к которому с помощью ключа К параллельно можно подключать другой конденсатор, как показано на рис. 1а.

При разомкнутом ключе К емкость на зажимах a и b равна С1, а при замкнутом ключе К эта емкость равна сумме С1+С2. Можно предположить, что переключение К из положения «1» в положение «2» с высокой частотой позволит регулировать емкость между зажимами a и b. Чем больше часть периода, в течение которой ключ К находится в положении «1», тем больше величина эквивалентной емкости Сэкв. Очевидно, что при изменении времени от 0 до Т, а g = t/T от 0 до 1, Сэкв будет изменяться в пределах от С1 до С1+С2.

Рассмотрим вначале идеальный случай, когда ключ К обладает нулевым сопротивлением (r = 0) в замкнутом состоянии и бесконечным сопротивлением (R = ∞) в разомкнутом состоянии. Кроме того, положим, что существует устройство, не показанное на рис. 1а, поддерживающее при разомкнутом ключе К напряжение на конденсаторе С2 равным напряжению на С1. Благодаря этому в момент замыкания ключа К никаких токов между конденсаторами С1 и С2, выравнивающих их напряжения, не возникает.

На временном интервале tn< t< tn+t ветвь ab описывается уравнением

откуда следует:

Аналогично на интервале tn+t< t< tn+1 = tn+Т получаем:

Средняя скорость изменения напряжения иab(t) на интервале tn< t< tn+1 равна

Если i(t) = I = const, то напряжение иab(t) будет изменяться по закону ломаной линии с чередующимися прямолинейными отрезками (рис. 1б). С уменьшением периода T ломаная линия все меньше будет отличаться от прямой (рис. 1б), имеющей наклон, равный средней скорости изменения напряжения иab(t):

Очевидно, что прямолинейный закон изменения, к которому реальный закон изменения напряжения иab(t) неограниченно приближается при T→0, имеет место при одном эквивалентном конденсаторе емкостью:

Таким образом, в идеальном случае при достаточно высокой частоте коммутации ключа К можно плавно изменять эквивалентную емкость на зажимах ab, изменяя относительную продолжительность подключения конденсатора С2g = t/Т. Эквивалентная емкость изменяется при этом в пределах от С1 при g = 0 до С1+С2 при g = 1.

В реальных условиях ключ К, реализованный на MOSFET или IGBT, имеет конечные сопротивления r 0 в открытом состоянии и R в закрытом состоянии. Использование устройства, поддерживающего при разомкнутом ключе К напряжение на конденсаторе С2 равным напряжению на конденсаторе С1, несколько уменьшает привлекательность рассматриваемого метода. Кроме того, при достаточно высокой частоте коммутации различие напряжений на конденсаторах С1 и С2 в момент замыкания ключа К может оказаться вполне допустимым с точки зрения ограничения уравнительного тока.

 

Анализ упрощенной схемы импульсного регулирования емкости конденсаторов

На временном интервале tn< t< tn+t эквивалентная схема, представленная на рис. 2а, описывается системой уравнений:

Приведение системы уравнений (5) к нормальной форме дает:

В векторно-матричной форме уравнения (6) записываются в виде:

где

Аналогично на временном интервале tn+t< t< tn+1 = tn+Т эквивалентная схема, представленная на рис. 2б, описывается системой дифференциальных уравнений (ДУ) в нормальной форме:

или в векторно-матричной форме:

где

Согласно основам теории систем с периодическим высокочастотным изменением структуры [3] предельная непрерывная модель системы описывается векторно-матричным уравнением:

где

Полученное описание предельной непрерывной модели системы (8) позволяет изобразить ее электрическую схему (рис. 2в). Таким образом, очевидно, что без поддержания при разомкнутом ключе К напряжения на конденсаторе С2 равным напряжению на конденсаторе С1, за счет изменения g изменяется только величина эквивалентного сопротивления rэкв = r/g. Поэтому при реально малой величине сопротивления r замкнутого ключа К и величине g, заметно превышающей нулевое значение, ветвь ав в цепи с сопротивлением, многократно превышающим сопротивление замкнутого ключа К, что всегда имеет место на практике, ведет себя практически как конденсатор емкостью С1+С2. Экспериментально этот вывод подтверждается исследованием процесса подключения ветви ав через сопротивление R, всего на два порядка превышающее сопротивление замкнутого ключа r, к источнику постоянного напряжения. Ток в цепи уменьшался практически по экспоненте с постоянной времени, равной R(С1+С2). Физически это объясняется тем, что увеличение напряжения на конденсаторе С1, при разомкнутом ключе К происходящее с большей скоростью, компенсируется уменьшением его за счет выравнивания напряжений при подключении конденсатора С2 в момент замыкания ключа К.

Таким образом, без поддержания напряжения на конденсаторе С2 равным при разомкнутом ключе К напряжению на конденсаторе С1эффективное импульсное управление величиной емкости осуществить невозможно. Следовательно, введение устройства, поддерживающего при разомкнутом ключе К напряжение на отключенном конденсаторе С2 равным напряжению на включенном конденсаторе С1, необходимо. Не останавливаясь на технической его реализации, имеющей очевидно множество вариантов, рассмотрим схему с управляемым генератором тока (ГТ), представленную на рис. 3а.

 

Предельная непрерывная модель схемы импульсного регулирования емкости конденсаторов

На первой части периода переключений К и К1tn< t< tn+t схема описывается уравнением (6а). Во второй части периода tn+t< t< tn+1 = = tn+Т справедлива система ДУ:

где ki коэффициент преобразования ГТ [ki] =А/В = Ом-1.

В векторно-матричной форме эта система уравнений имеет вид:

где

Предельная непрерывная модель схемы рис. 3а описывается уравнением:

где

Система ДУ предельной непрерывной модели системы имеет вид:

Подставляя (uC1uC2), выраженное из второго уравнения, в первое и учтя, что uC1 ~ uC2, с хорошей точностью получаем:

 

где

 

При kir = 1 получаем

Сэкв = С1+ 2. (12а)

Переходя к изображениям по Лапласу в векторно-матричном уравнении (10) при нулевых начальных условиях, получаем:

Uab(p) = UC1(p) = cT[pEA]-1hI(p),

где Uab(p) = UC1(p) = L{uC1(t)}, I(p) = L{i(t)}— преобразование Лапласа uC1(t) и i(t) соответственно, Е — единичная 2×2 матрица, cT = [1,0] — вектор-строка.

Из последнего уравнения получаем передаточную функцию предельной непрерывной модели цепи рис. 3а:

Разложение W(p) на сумму двух простейших дробей дает:

где

Согласно (14) можно построить эквивалентную электрическую схему предельной непрерывной модели, представленную на рис. 3б:

Переходя к изображениям по Лапласу, получаем:

Сравнение выражений (14) и (15) дает:

 

Экспериментальная проверка полученных теоретических результатов

С целью экспериментальной проверки полученных теоретически результатов моделировалось подключение RC-цепи к генератору синусоидального напряжения частотой f = 50 Гц, амплитудой Um = 100 В и нулевой начальной фазой j = 0. Принято R = 10 Ом, а величина Сустанавливалась равной 1,2×10-4 и 1,8×10-4 Ф путем импульсного регулирования по схеме рис. 3а. При этом С1 = С2 =1×10-4 Ф, T = 10-5 с, r = 0,1 Ом, kir = 1. В соответствии с формулой (12а) величина g принималась равной 0,2 и 0,8 соответственно при Сэкв = 1,2×10-4 и 1,8×10-4Ф.

Одновременно моделировалось подключение к тому же напряжению RC-цепи, в которой импульсно регулируемый конденсатор представлялся его предельной моделью рис. 3б. Схема, построенная в системе моделирования MATLAB 6.5 Simulink 5 Sim Power System, изображена на рис. 4.

Рис. 4. Схема моделирования в системе MATLAB 6.5 Simulink 5 Sim Power System

Результаты моделирования представлены на рис. 5а, б. На временных диаграммах i — ток RC-цепи с регулированием емкости конденсатора, iМ ток RC-цепи, в которой регулируемый конденсатор представлен его предельной непрерывной моделью, Di = iМ – i.

Рис. 5. Временные диаграммы токов в реальной схеме и в ее предельной непрерывной модели (i и iM) и их разности, увеличенной в 100 раз: а) Сэкв = 1,2×10-4 Ф; б) Сэкв = 1,8×10-4 Ф

Моделирование проводилось методом ode15s при ограничении максимального шага интегрирования величиной 10-6.

Анализ результатов моделирования показывает очень хорошее приближение предельно непрерывной модели к реальному импульсно регулируемому конденсатору. Увеличение T снижает точность приближения. Следует подчеркнуть, что рассмотренный способ регулирования емкости конденсаторов представляет собой схемное решение. Емкости конденсаторов в действительности не изменяются. В момент уменьшения емкости ветви ab (в момент отключения С2) напряжение на ветви не изменяется, хотя при мгновенном уменьшении емкости конденсатора напряжение на нем (uC = q/C) возрастает.

 

Выводы

  • Изменение относительной продолжительности периодического подключения дополнительного конденсатора параллельно основному при поддержании на отключенном конденсаторе напряжения, равного напряжению на основном конденсаторе, позволяет плавно регулировать эквивалентную емкость.
  • При достаточно высокой частоте переключений схему регулирования емкости можно с необходимой точностью заменить ее эквивалентной предельной непрерывной моделью.
Литература
  1. Баскаков С. И. Радиотехнические цепи и сигналы. М: Высшая школа. 2003.
  2. Торопцев Н. Д. Авиационные асинхронные генераторы. М: Транспорт. 1970.
  3. Коршунов А. И. Предельная непрерывная модель системы с высокочастотным периодическим изменением структуры // Изв. вузов. Приборостроение. 2009. Т. 52, № 9.

Электрическая емкость (страница 1)

Решение:
При перемещении пластины емкость конденсатора в данный момент времени определяется той частью площади пластин, по которой они перекрывают друг друга. В моменты времени t1 и t2 площади

где l=10 см-длина стороны пластины. В эти моменты времени конденсатор имеет емкости

а заряды на его пластинах

11 Найти заряд, который нужно сообщить двум параллельно соединенным конденсаторам с емкостями C1 = 2 мкФ и С2=1 мкФ, чтобы зарядить их до разности потенциалов V=20кВ.

Решение:
Общий заряд параллельно соединенных конденсаторов

12 Два одинаковых плоских конденсатора соединены параллельно и заряжены до разности потенциалов V0 = 6 В. Найти разность потенциалов V между пластинами конденсаторов, если после отключения конденсаторов от источника тока у одного конденсатора уменьшили расстояние между пластинами вдвое.

Решение:

13 Два конденсатора с емкостями С1 = 1 мкФ и С2 = 2мкФ зарядили до разностей потенциалов V1=20B и V2 = 50 В. Найти разность потенциалов V после соединения — конденсаторов одноименными полосами.

Решение:



14 Конденсатор емкости C1 = 20 мкФ, заряженный до разности потенциалов V1 = 100B, соединили параллельно с заряженным до разности потенциалов V1=40 В конденсатором, емкость которого С2 неизвестна (соединили одноименно заряженные обкладки конденсаторов). Найти емкость С2 второго конденсатора, если разность потенциалов между обкладками конденсаторов после соединения оказалась равной V=80 В.

Решение:

15 Конденсатор емкости С1=4мкФ, заряженный до разности потенциалов V1 = 10B, соединен параллельно с заряженным до разности потенциалов V2 = 20 В конденсатором емкости С2 = 6 мкФ (соединили разноименно заряженные обкладки конденсаторов). Какой заряд окажется на пластинах первого конденсатора после соединения?

Решение:
Заряды конденсаторов до их соединения q1 = C1V1 и q2 = C2V2. После соединения разноименно заряженных обкладок конденсаторов общий заряд q = |q2-q1| = (C1 + C2)V и заряд первого конденсатора где V-разность потенциалов между обкладками конденсаторов после соединения; отсюда

16 Конденсатор, заряженный до разности потенциалов V1 = 20 В, соединили параллельно с заряженным до разности потенциалов V2 = 4 В конденсатором емкости С2 = 33 мкФ (соединили разноименно заряженные обкладки конденсаторов). Найти емкость С1 первого конденсатора, если разность потенциалов между обкладками конденсаторов после их соединения V=2 В.

Решение:
После соединения разноименных обкладок общий заряд q = CV равен разности зарядов q1 = C1V1 и q2 = C2V2 отдельных конденсаторов, где С=С1 + С2 — общая емкость после соединения. Таким образом,


17 Конденсатор емкости С1 = 1 мкФ, заряженный до разности потенциалов V1 = 100B, соединили с конденсатором емкости С2 = 2 мкФ, разность потенциалов V2 на обкладках которого неизвестна (соединили разноименно заряженные обкладки конденсаторов). Найти разность потенциалов V2, если разность потенциалов между обкладками конденсаторов после соединения оказалась равной V=200 В.

Решение:
До соединения заряды первого и второго конденсаторов

После соединения разноименных обкладок общий заряд

Двойной знак мы здесь поставили потому, что заранее не известно, какой из зарядов, q2 или q1 больше; отсюда

Решение со знаком минус соответствует случаю, когда знаки зарядов на пластинах первого конденсатора после соединения пластин не меняются, а со знаком плюс-случаю, когда эти знаки становятся обратными. Так как в нашем случае , а величина |V2| должна быть всегда положительной, то существует лишь одно решение-со знаком плюс. В результате |V2| = 350 В.

18 Два проводящих шара с радиусами R1 и R2 расположены так, что расстояние между ними во много раз больше радиуса большего шара. На шар радиуса R1 помещен заряд q. Каковы будут заряды на шарах после соединения их проводником, если второй шар не был заряжен? Емкостью проводника, соединяющего шары, пренебречь.

Решение:


19 Два проводящих шара с радиусами R1 = 8см и R2 = 20 см, находящихся на большом расстоянии друг от друга, имели электрические заряды q1=40 нКл и q2=— 20 нКл. Как перераспределятся заряды, если шары соединить проводником? Емкостью проводника, соединяющего шары, пренебречь.

Решение:
Соединение шаров проводником эквивалентно параллельному соединению конденсаторов. После соединения


20 Два проводящих шара с радиусами R1 = 10см и R2 = 5см, заряженных до потенциалов φ1=20B и φ2=10В, соединяются проводником. Найти поверхностные плотности зарядов на шарах σ1 и σ2 после их соединения. Расстояние между шарами велико по сравнению с их радиусами. Емкостью проводника, соединяющего шары, пренебречь.

Решение:
Заряды на шарах до и после соединения Общий потенциал шаров после соединения определим из условия сохранения заряда
Заряды на первом и втором шарах после соединения

Поверхностные плотности зарядов на шарах


21 Плоский воздушный конденсатор, заряженный до разности потенциалов V0 = 800 В, соединили параллельно с таким же по размерам незаряженным конденсатором, заполненным диэлектриком. Какова диэлектрическая проницаемость e диэлектрика, если после соединения разность потенциалов между пластинами конденсаторов оказалась равной V=100В?

Решение:

22 Найти емкость С трех плоских воздушных конденсаторов, соединенных параллельно. Размеры конденсаторов одинаковы: площадь пластины S=314 см2, расстояние между пластинами d=1 мм. Как изменится емкость трех конденсаторов, если пространство между пластинами одного конденсатора заполнить слюдой (диэлектрическая проницаемость ε1 = 7), а другого — парафином (диэлектрическая проницаемость ε2 = 2)?

Решение:
Емкость трех конденсаторов без диэлектрика При заполнении двух конденсаторов диэлектриками емкость трех конденсаторов

23 В заряженном плоском конденсаторе, отсоединенном от источника тока, напряженность электрического поля равна Е0. Половину пространства между пластинами конденсатора заполнили диэлектриком с диэлектрической проницаемостью ε (толщина диэлектрика равна расстоянию между пластинами). Найти напряженность электрического поля Е в пространстве между пластинами, свободном от диэлектрика.

Решение:
Если d-расстояние между пластинами и С0 — емкость конденсатора без диэлектрика, то разность потенциалов между пластинами конденсатора (без диэлектрика) и заряд на пластинах Конденсатор, половина которого заполнена диэлектриком, можно рассматривать как два соединенных параллельно конденсатора (рис. 341), причем один не содержит диэлектрика и имеет емкость а в другом все пространство между пластинами заполнено диэлектриком, и поэтому его емкость Полная емкость конденсатора, половина которого заполнена диэлектриком, При отключенном источнике тока заряд на пластинах сохраняется, поэтому разность потенциалов между пластинами V=q/C, и напряженность электрического поля в пространстве между пластинами, свободном от диэлектрика,


24 Два последовательно соединенных конденсатора с емкостями C1 = 1 мкФ и С2 = 3 мкФ подключены к источнику тока с напряжением V =220 В. Найти напряжение на каждом конденсаторе.

Решение:
Если V1 и V2 — напряжения на первом и втором конденсаторах, то V= V1 + V2, а заряды на них одинаковы и равны
q=C1V1=C2V2; отсюда

При последовательном соединении конденсаторов на конденсаторе меньшей емкости напряжение больше, чем на конденсаторе большей емкости.

25 Два последовательно соединенных конденсатора с емкостями C1 = 1 мкФ и С2 = 2 мкФ подключены к источнику тока с напряжением V =900 В. Возможна ли работа такой схемы, если напряжение пробоя конденсаторов Vпр = 500 В?

Решение:
Напряжения на первом и втором конденсаторах (см. задачу 24). Работать при указанном в условии задачи напряжении пробоя конденсаторов нельзя, ибо произойдет пробой первого, а затем и второго конденсаторов.

26 Два последовательно соединенных конденсатора подключены к источнику тока с напряжением V= 200 В (рис. 79). Один конденсатор имеет постоянную емкость C1 = 0,5 мкФ, а другой — переменную емкость С2 (от Cmin = 0,05 мкФ до Сmах = 0,5 мкФ). В каких пределах изменяется напряжение на переменном конденсаторе при изменении его емкости от минимальной до максимальной?

Решение:
При изменении емкости переменного конденсатора С2 от Cmin до Сmax, напряжение на нем V изменяется в пределах (см. задачу 24)

27 При последовательном соединении трех различных конденсаторов емкость цепи С0 = 1 мкФ, а при параллельном соединении емкость цепи С=11мкФ. Найти емкости конденсаторов С2 и С3, если емкость конденсатора С1 = 2 мкФ.

Решение:

28 При последовательном соединении трех различных конденсаторов емкость цепи С0 = 0,75 мкФ, а при параллельном соединении емкость цепи С = 7 мкФ. Найти емкости конденсаторов С2 и С3 и напряжения на них V2 и V3 (при последовательном соединении), если емкость конденсатора C1 = 3 мкФ, а напряжение на нем V1 = 20B.

Решение:
При последовательном соединении конденсаторов имеем

при параллельном

Из этих уравнений находим

Согласно теореме Виета С2 и С3 должны быть корнями квадратного уравнения

Решая его, найдем

Заряды на всех конденсаторах при последовательном соединении равны между собой:


29 Три последовательно соединенных конденсатора с емкостями С1 = 100пФ, С2 = 200 пФ, С3 = 500 пФ подключены к источнику тока, который сообщил им заряд q=10нКл. Найти напряжения на конденсаторах V1, V2 и V3, напряжение источника тока V и емкость всех конденсаторов С0.

Решение:
При последовательном соединении конденсаторов заряд каждого конденсатора равен q, поэтому

Напряжение источника тока равно полному напряжению на всех конденсаторах:

Так как при последовательном соединении
то

30 Три последовательно соединенных конденсатора с емкостями С1=0,1мкФ, С2 = 0,25 мкФ и С3 = 0,5 мкФ подключены к источнику тока с напряжением V =32 В. Найти напряжения V1, V2 и V3 на конденсаторах.

Решение:


31 Два одинаковых воздушных конденсатора емкости С=100пФ соединены последовательно и подключены к источнику тока с напряжением V= 10 В. Как изменится заряд на конденсаторах, если один из них погрузить в диэлектрик с диэлектрической проницаемостью ε = 2?

Решение:
При последовательном соединении конденсаторов заряды на конденсаторах равны. До погружения одного из них в диэлектрик заряд на каждом конденсаторе

после погружения одного из них в диэлектрик заряды конденсаторов будут

Учитывая, что

Изменение заряда на конденсаторах


32 Два плоских воздушных конденсатора с одинаковыми емкостями соединены последовательно и подключены к источнику тока. Пространство между пластинами одного из конденсаторов заполняют диэлектриком с диэлектрической проницаемостью ε = 9. Во сколько раз изменится напряженность электрического поля Е в этом конденсаторе?

Решение:
Первоначальная напряженность электрического поля в каждом конденсаторе

где d-расстояние между пластинами конденсатора. После заполнения одного конденсатора диэлектриком напряженность электрического поля в нем

Отношение напряженностей


33 Решить предыдущую задачу для случая, когда конденсаторы после зарядки отключаются от источника тока.

Решение:
После отключения конденсатора от источника тока и заполнения его диэлектриком заряд на нем не изменяется:

Напряженность электрического поля в конденсаторе, заполненном диэлектриком,

Отношение напряженностей

34 Два плоских воздушных конденсатора с одинаковыми емкостями С=10пФ соединены последовательно. Насколько изменится емкость конденсаторов, если пространство между пластинами одного из них заполнить диэлектриком с диэлектрической проницаемостью ε = 2?

Решение:
Изменение емкости соединенных конденсаторов


35 В плоский воздушный конденсатор с площадью обкладок S и расстоянием между ними d введена параллельно обкладкам проводящая пластинка, размеры которой равны размерам обкладок, а ее толщина намного меньше d. Найти емкость конденсатора с проводящей пластинкой, если пластинка расположена на расстоянии l от одной из обкладок конденсатора.

Решение:
После введения пластинки образовалось два последовательно включенных конденсатора с емкостями

(рис. 342). Их общую емкость определим из соотношения

где С-первоначальная емкость конденсатора. Таким образом, после введения пластинки при любом ее положении С0 = С.

36 В плоский воздушный конденсатор с площадью обкладок S и расстоянием между ними d введена параллельно обкладкам проводящая пластинка, размеры которой равны размерам обкладок, а толщина dп = d/3

Решение:
Введение проводящей пластинки между обкладками конденсатора приводит к образованию двух последовательно включенных конденсаторов с расстояниями между обкладками d1 и d2 и емкостями

(рис.343). Их общую емкость находим из соотношения

При -первоначальная емкость конденсатора.

37 Плоский воздушный конденсатор заряжен до разности потенциалов V0 = 50 В и отключен от источника тока. После этого в конденсатор параллельно обкладкам вносится проводящая пластинка толщины dп= 1 мм. Расстояние между обкладками d=5 мм, площади обкладок и пластинки одинаковы. Найти разность потенциалов V между обкладками конденсатора с проводящей пластинкой.

Решение:
Емкости конденсатора до и после внесения проводящей пластинки толщины dп (см. задачу 36)
Заряд конденсатора, отключенного от источника тока, не изменяется:

отсюда разность потенциалов между обкладками конденсатора после внесения проводящей пластинки

38 В плоский воздушный конденсатор с площадью обкладок S и расстоянием между ними d вводится параллельно обкладкам диэлектрическая пластинка толщины d1<d/ Диэлектрическая проницаемость пластинки равна ε, площади обкладок и пластинки одинаковы и равны S. Найти емкость конденсатора с диэлектрической пластинкой.

Решение:

При изменении емкости конденсатора на 100 пФ резонансная частота

Условие задачи:

При изменении емкости конденсатора на 100 пФ резонансная частота колебательного контура увеличилась от 0,2 до 0,25 МГц. Какой индуктивностью обладает контур?

Задача №9.12.5 из «Сборника задач для подготовки к вступительным экзаменам по физике УГНТУ»

Дано:

\(\Delta C = 100\) пФ, \(\nu_1=0,2\) МГц, \(\nu_2=0,25\) МГц, \(L-?\)

Решение задачи:

Резонансная частота равна собственной частоте колебаний этого колебательного контура, поэтому её можно определить по формуле:

\[\nu = \frac{1}{{2\pi \sqrt {LC} }}\;\;\;\;(1)\]

В этой формуле \(L\) – индуктивность катушки, \(C\) – электроемкость конденсатора.

Из формулы (1) мы видим, что зависимость между электроемкостью контура и резонансной частотой контура обратно пропорциональная. Учитывая это, запишем две формулы для определения частот \(\nu_1\) и \(\nu_2\):

\[\left\{ \begin{gathered}
{\nu _1} = \frac{1}{{2\pi \sqrt {LC} }} \;\;\;\;(2)\hfill \\
{\nu _2} = \frac{1}{{2\pi \sqrt {L\left( {C – \Delta C} \right)} }} \hfill \\
\end{gathered} \right.{ – 3}}\;Гн = 2,3\;мГн\]

Ответ: 2,3 мГн.

Если Вы не поняли решение и у Вас есть какой-то вопрос или Вы нашли ошибку, то смело оставляйте ниже комментарий.

M6013 Портативный цифровой измеритель емкости конденсатора

Особенности:

— Увеличенный диапазон измерения, считывание до 470 мФ / 470000uF

— с функцией автоматического усреднения работы, может читать pF более стабильно и точно

— Простое в использовании и быстрое время отклика при измерении от 0,01 п.н. до 470000 мкФ

Заметка:

1. Точность может зависеть от длины испытательного провода и расстояния от измерительных проводов. Рекомендуется проводить тестирование небольшой емкости pF, кратчайший тестовый провод, и будьте осторожны, окружающие электромагнитные помехи или радиочастотный шум могут повлиять на чтение pF.

2. ПОЖАЛУЙСТА, РАССМАТРИВАЙТЕ ПАРАКТИКОР ПЕРЕД ИСПЫТАНИЕМ, вы можете использовать отвертку для короткого замыкания или серии с резистором около 10 Ом и коротким от 5 до 10 секунд. Внутри измерителя есть 1 быстрый стабилитрон, чтобы предотвратить высокое напряжение, но важно сначала разрядить конденсатор, так как это большой импульсный ток и напряжение и может повредить счетчик.

Содержание пакета:

1 x цифровой измеритель емкости

1 x пара тестового зонда

Введение:

— ВКЛЮЧИТЬ

Нажмите и удерживайте кнопку включения / выключения круглой оранжевой кнопки в течение 1 — 2 секунд для включения.

Нажмите и отпустите кнопку ON / OFF, чтобы выключить счетчик.

— Режим AUTO / MANUAL:

Автодиапазон:

Нажмите и отпустите кнопку RANGE, и на первой строке ЖК-дисплея отобразится «AUTO:»

В автоматическом режиме счетчик автоматически выбирает наилучший диапазон для обнаружения.

Ручной диапазон:

Прокрутите ручной диапазон от диапазонов 47nF, 47uF и 470mF, нажав и отпустите кнопку RANGE

На ЖК-дисплее отобразится MANUAL на первой ЖК-линии и отобразится на второй строке: 0-47NF, 47NF до 47UF и 47UF до 470MF.

1 Ключ:

OPEN подключите клемму испытательного провода.

Нажмите и отпустите кнопку «ZERO», ЖК-дисплей отобразит «ZERO» и подождите, пока не исчезнет нуль.

Если вы используете сокет измерительной решетки, вам нужно также открыть цепь OPEN для установки нуля.

Разъем:

Вы можете использовать 8-контактный разъем для измерения конденсатора

Средний:

Он автоматически начнет работать средним, если показание емкости станет стабильным, и пользователь сможет получить более точное считывание.

Во время работы он покажет «AVG» на правом нижнем ЖК-дисплее или отобразит значок необработанных данных «RAW»,

В результате, если вы хотите получить более высокую точность чтения, вы можете взять показания во время отображения AVG.

Эта функция автоматически включается, когда исходные данные являются помехами.

Подсветка:

Подсветка ЖК-дисплея будет включена во время включения

Авто спящий режим:

Около 8-10 часов для не тестирования, он автоматически отключится для экономии энергии.

OVERFLOW или OL:

Переполнение дисплея, когда значение вне диапазона, вы можете проверить, что вы равны нулю.

факторов, влияющих на емкость | Конденсаторы

Существует три основных фактора конструкции конденсатора, определяющих величину создаваемой емкости. Все эти факторы определяют емкость, влияя на то, какой поток электрического поля (относительная разница электронов между пластинами) будет развиваться для данной величины силы электрического поля (напряжения между двумя пластинами):

ПЛОЩАДЬ ПЛАСТИНЫ : При прочих равных условиях большая площадь пластины дает большую емкость; меньшая площадь пластины дает меньшую емкость.

Пояснение: Большая площадь пластины приводит к большему потоку поля (заряд, собранный на пластинах) для данной силы поля (напряжение на пластинах).

РАССТОЯНИЕ ПЛАСТИН : При прочих равных условиях большее расстояние между пластинами дает меньшую емкость; меньшее расстояние между пластинами дает большую емкость.

Пояснение: Более близкое расстояние приводит к большей силе поля (напряжение на конденсаторе, деленное на расстояние между пластинами), что приводит к большему потоку поля (заряд, накопленный на пластинах) для любого заданного напряжения, приложенного к пластинам.

ДИЭЛЕКТРИЧЕСКИЙ МАТЕРИАЛ : При прочих равных условиях большая диэлектрическая проницаемость диэлектрика дает большую емкость; меньшая диэлектрическая проницаемость диэлектрика дает меньшую емкость.

Пояснение: Хотя это сложно объяснить, некоторые материалы предлагают меньшее сопротивление потоку поля для данной величины силы поля. Материалы с большей диэлектрической проницаемостью допускают больший поток поля (предлагают меньшее сопротивление) и, следовательно, больший накопленный заряд для любой заданной величины силы поля (приложенного напряжения).

«Относительная» диэлектрическая проницаемость означает диэлектрическую проницаемость материала относительно диэлектрической проницаемости чистого вакуума. Чем больше число, тем больше диэлектрическая проницаемость материала. Стекло, например, с относительной диэлектрической проницаемостью 7, имеет в семь раз большую диэлектрическую проницаемость чистого вакуума и, следовательно, позволяет создать поток электрического поля, в семь раз более сильный, чем у вакуума, при прочих равных условиях. В следующей таблице перечислены относительные диэлектрические проницаемости (также известные как «диэлектрическая проницаемость») различных распространенных веществ:

Материал

Относительная диэлектрическая проницаемость (диэлектрическая проницаемость)
Вакуум 1.0000
Воздух 1.0006
PTFE, FEP («тефлон») 2,0
Полипропилен от 2,20 до 2,28
Смола АБС от 2,4 до 3,2
Полистирол от 2,45 до 4,0
Вощеная бумага 2,5
Масло трансформаторное 2,5 до 4
Твердая резина 2.От 5 до 4,80
Дерево (Дуб) 3,3
Силиконы от 3,4 до 4,3
Бакелит от 3,5 до 6,0
Кварц плавленый 3,8
Дерево (клен) 4,4
Стекло от 4,9 до 7,5
Касторовое масло 5,0
Дерево (береза) 5,2
Слюда, мусковит 5.От 0 до 8,7
Стекловолоконная слюда от 6,3 до 9,3
Фарфор, стеатит 6,5
Глинозем от 8,0 до 10,0
Вода дистиллированная 80,0
Барий-стронций-титанит 7500

Примерную емкость для любой пары разделенных проводов можно найти по следующей формуле:

Конденсатор можно сделать переменным, а не фиксированным, путем изменения любого из физических факторов, определяющих емкость.Один относительно простой фактор, который можно изменить в конструкции конденсатора, — это площадь пластины или, точнее, величина перекрытия пластин.

На следующей фотографии показан пример переменного конденсатора, использующего набор чередующихся металлических пластин и воздушный зазор в качестве диэлектрического материала:

При вращении вала степень перекрытия наборов пластин будет изменяться, изменяя эффективную площадь пластин, между которыми может быть установлено концентрированное электрическое поле.Этот конкретный конденсатор имеет емкость в пикофарадном диапазоне и находит применение в радиосхемах.

СВЯЗАННЫЕ РАБОЧИЕ ЛИСТЫ:

Конденсаторы

— Емкость, диполи и диэлектрическое поглощение — Блог о пассивных компонентах

C1.1 МОЩНОСТЬ

Емкость определяется, среди прочего, характеристиками диэлектрического материала. Международные стандарты говорят о диэлектрической постоянной или диэлектрической проницаемости , обозначенной символом ε.


C1.1.1 Описание

Конденсатор служит резервуаром для электрических зарядов. Размер «резервуара» называется емкостью и выражается величиной F (арад) или As / V. Принципиальный рисунок C1-1 показывает, как емкость прямо пропорциональна активной площади A и диэлектрической проницаемости и обратно пропорциональна расстоянию между электродами. Формула на рисунке применима к вакууму и воздуху.

A = площадь (м 2 ),

d = расстояние между электродами (м),

ε 0 = диэлектрическая проницаемость для вакуума (≈air) = 1 × 10 -9 / 36π.

Рисунок C1-1. Принцип емкости, кл.

Если величина электрического заряда конденсатора обозначена Q (As), то действует общая формула C1-1.

……………………………… [C1-1]

Рисунок C1-2. Диэлектрик с его постоянной.

Если мы теперь вставим изоляционный материал между электродами, как показано на рисунке C3-2, формула на рисунках C3-1 и -2 получит следующее общее выражение

……………………………….[C1-2]

ε r — это относительное число — относительная диэлектрическая проницаемость — которое говорит нам, во сколько раз увеличивается емкость, когда мы заменяем воздушный зазор между электродами из разных диэлектрических материалов. Это относительная диэлектрическая проницаемость ε r , которая указана в технических таблицах и каталогах.

В таблице ниже показана диэлектрическая проницаемость наиболее распространенных материалов.

Рисунок C1-2b Диэлектрическая проницаемость наиболее распространенных изоляционных материалов

Емкостное реактивное сопротивление

Если мы изменим полярность на рисунке C1-2, приложив переменное напряжение к конденсатору, это вызовет определенное сопротивление в цепи, так называемое емкостное реактивное сопротивление, X C , выраженное в омах.

Реактивное сопротивление обратно пропорционально частоте согласно формуле

…………………… .. [C1-3]

  • ω = 2 x π x f,
  • f = частота в Гц,
  • C = емкость в F.

Мера миниатюризации

Желаемая миниатюризация конденсаторов разных типов может быть выражена по-разному. Наименьшего номинального напряжения для электростатических конденсаторов часто более чем достаточно для применения, как показано в разделе C2.1.4 тоже. Следовательно, мы обычно не обращаем внимания на напряжение и сравниваем различные типы с помощью их максимально возможной скорости C / V, что означает емкость C на единицу объема V (d * A на рисунке C1-1). По формуле C1-2 получаем C / V = ​​ε 0 * ε r * A / (d * A * d) = ε 0 * ε r / d 2 . Скорость C / V будет максимальной для d мин , то есть для V Rmin .

В электролитических конденсаторах номинальное напряжение играет большую роль, потому что оно может быть адаптировано также к очень низким рабочим напряжениям.Здесь конденсаторы сгруппированы в соответствии с их количеством заряда, то есть C R * V R . Мы ссылаемся на продукт CV .

Подключения

Если мы подключим конденсаторы параллельно , как показано на рисунке C1-3, активная область (и, следовательно, емкость) увеличится со всеми дополнительными элементами конденсатора.

Рисунок C1-3. Принципиальная величина полной емкости при параллельном соединении элементов.

Формула для общей емкости параллельных соединений :

……………. [C1-4]

На рисунке C1-4 в принципе показано, как соединение последовательно увеличивает толщину диэлектрика без изменения количества заряда. Емкость уменьшается пропорционально увеличению толщины диэлектрика.

Рисунок C1-4. Принцип последовательного подключения.

На рисунке показано упрощенное изображение последовательного соединения с двумя одинаковыми по размеру конденсаторами.Если вместо этого мы выберем конденсаторные элементы разного размера, количество заряда на разных электродах все равно будет одинаково большим, то есть Q = CV = C 1 В 1 = C 2 В 2 = C 3 В 3 ; Q / C = V; Q / C 1 = V 1 ; Q / C 2 = V 2 ; Q / C 3 = V 3… Но V 1 + V 2 + V 3 +…. = V. Это дает нам в целом общую емкость для
последовательного соединения :

………….[C1-5]

Смешанные диэлектрики

Все чаще встречаются так называемые смешанные диэлектрики. Они состоят из разных пленочных материалов в одном конденсаторе. Например, намотав конденсатор как из бумаги, так и из диэлектрика из полиэфирной пленки, мы сочетаем превосходные самовосстанавливающиеся свойства бумаги и относительно высокое сопротивление изоляции полиэстера. В принципе, это все еще вопрос о двух последовательно соединенных элементах конденсатора с одинаковой площадью и толщиной диэлектрика d 1 + d 2 .Тогда верно изображенное выше соотношение: Q = VC = V 1 C 1 = V 2 C 2 ; V 1 x ε 1 x A / d 1 = V 2 x ε 2 x A / d 2 ; если мы денонсируем напряженность электрического поля E, то получим
ε 1 x E 1 = ε 2 x E 2 , или в целом

ε 1 x E 1 = ε 2 x E 2 = ε 3 x E 3 = ……………………… [C1-6]

Прочие вычисления емкости без пластин и емкости с различной геометрией

Расчет емкости по уравнению [C1-2] и показанный на рисунке C1-2 основан на типе плоского конденсатора.Однако на рынке существует множество других типов конструкции и геометрии конденсаторов. См. Расчет теоретического значения емкости для некоторых других конфигураций, а также для смешанных диэлектрических ситуаций ниже:


Стандартизированные значения емкости и допуск

В соответствии с международными стандартами EIA / IEC 62 значения емкости и допуски стандартизированы следующим образом:

Диапазон E

Емкость соответствует стандартизированным «диапазонам E», определенным для логарифмических шагов, таких как шаги E3, E6… E24, E48.

Естественно, выбранный диапазон E также связан с полем допуска — чтобы не перекрывать следующий диапазон допуска емкости — см. Ниже.

Конденсаторные технологии серии E определяются его способностью производить воспроизводимые значения емкости с жесткими допусками при массовом производстве. Вы можете найти соответствующие значения емкости и диапазоны допусков, определенные в каталогах производителей.


C 1.1.2 Диполи

Раздел о диполях и диэлектрическом поглощении имеет жизненно важное значение для понимания практического конденсатора.Все материалы содержат какие-то диполи, то есть электрически полярные элементы. Когда они подвергаются воздействию электрического поля, он создает крутящий момент, который, в зависимости от напряженности поля, будет стремиться выровнять их в этом поле. Эти крутящие моменты можно разделить на четыре группы. Те, которые вызваны

  • движений электронов в атомах и молекулах,
  • движений атомов в симметричных молекулах,
  • движений атомов в несимметричных молекулах и
  • накоплений заряда на границах раздела между различными материалами в диэлектрике.

Пока конденсатор не смещен, диполи имеют произвольную ориентацию без какого-либо результирующего полюса. В принципе это может выглядеть как на рисунке C1-5.

Рисунок C1-5. Ориентация диполя в несмещенном диэлектрике.

Если они должны быть подвергнуты воздействию напряженности электрического поля, как показано на рисунке C1-6, через определенное время они будут стремиться выровняться в дипольные цепи. Материал диэлектрика — поляризованный .

Рисунок C1-6.Идеально выровненные дипольные цепи.

Напряженность электрического поля (количество воображаемых силовых линий, которые могли бы образоваться в вакууме) была уменьшена с увеличением количества установленных дипольных цепочек. Каждая дипольная цепь связывает на границе раздела, например, с положительным электродом + заряд, и количество свободных носителей заряда в электроде было уменьшено до соответствующей степени. Таким образом, по истечении времени настройки диполей электрод может принять столько новых свободных носителей заряда, сколько тех, которые связали дипольные цепи , без увеличения напряженности электрического поля (или напряжения) по сравнению с исходным напряжением. точка.Это означает соответствующее увеличение емкости. Если мы назовем эту поляризуемость , количество связанных зарядов q и количество зарядов в начальной точке Q , можно показать, что

……………………. [C1-7]

Зависимость частоты от частоты показана на Рисунке C1-8 ниже.

Поскольку r в зависимости от материала диэлектрика варьируется примерно от двух до многих тысяч, мы понимаем, какое огромное значение играют диполи материала и поляризуемость.

Частотная зависимость емкости

Скорость, с которой диполь реагирует на приложенное электрическое поле, называется временем его релаксации. Эти времена релаксации составляют от 10 -17 с для электронно-зависимых диполей до нескольких часов для больших молекулярных комплексов. Это означает, что самые быстрые диполи успевают за всеми практическими частотами, в то время как более медленные в той или иной степени требуют времени, чтобы внести свой вклад в дипольные цепи, увеличивающие емкость.Это явление можно описать как основной конденсатор в сочетании с рядом дополнительных конденсаторных элементов, скрытых в резистивных цепях с более короткими или более длинными постоянными времени (рисунок C1-7).

Рисунок C1-7. Дипольные категории в конденсаторе.

Пример частотного диапазона, в который вносят вклад различные типы диполей, показан на рисунке C1-8.

  • α e = дипольный эффект от движений электронов;
    • α a = дипольный эффект от движений атомов в симметричных диполях;
  • α d = дипольный эффект от движений атомов в несимметричных молекулах;
  • α i = диполи, зависящие от границы раздела фаз.

Рисунок C1-8. Типичный пример схематического изменения поляризуемости твердого материала от частоты.

Таким образом, емкость уменьшается с увеличением частоты. В компонентах с большими диэлектрическими потерями и значительным процентом инертных диполей мы узнаем, как кривая импеданса начинает отклоняться от номинальной кривой емкостного реактивного сопротивления, когда мы приближаемся к резонансной частоте. Рисунок C1-20.


C 1.1.3 Диэлектрическое поглощение

Если диполи были «активированы» для образования дипольной цепи, потребуется соответствующее время, чтобы «деактивировать» их при той же температуре.На рисунке C1-9 предполагается, что конденсатор сначала был заряжен, затем на мгновение закорочен и, наконец, оставлен открытым. Те дипольные цепи, которые были слишком инертны, чтобы реагировать в момент короткого замыкания, удерживали свои заряды в электроде. Через некоторое время в отсутствие электрического поля они начинают принимать случайные, невыровненные положения, высвобождая захваченные заряды в электродах (рис. C1-9). Высвободившиеся заряды проявляются как остаточное напряжение в конденсаторе и измеряются в В.Это остаточное напряжение является мерой диэлектрического поглощения «DA» конденсатора и выражается в процентах от приложенного начального напряжения.

Рисунок C1-9. Эффект диэлектрического поглощения.

DA обычно является нежелательным свойством, которое сильно нагружает одни диэлектрические материалы, другие — незначительно или совсем незначительно. Иногда это может вызвать проблемы, которые мы обсудим позже.

Определение DA производится путем смещения конденсатора напряжением постоянного тока в течение определенного периода времени, затем короткого замыкания части через резистор на указанное количество секунд и, наконец, оставления его разомкнутым на несколько минут до остаточного напряжения. читается.Выражается в процентах от зарядного напряжения. Напряжение, время и сопротивление указаны в разных стандартах, которые иногда различаются. Примеры того, как время влияет на результаты, показаны в Таблице C3-1, где записи ведутся при 25 ° C. DA сильно увеличивается с повышением температуры.

Таблица C1-1. Примеры DA при 25 ° C

Знание диэлектрического поглощения конденсаторов часто является жизненно важным для оптимального проектирования схемы. Таким образом, мы предоставим значения DA в кратком описании деталей после каждой группы материалов.В первую очередь, значения получаются на основе процедур, аналогичных или равных любому из методов в Таблице C1-1 выше.

См. Ниже таблицу диэлектрического поглощения наиболее распространенных конденсаторных технологий:


ABC CLR: Глава C Конденсаторы

Емкость, диполи и диэлектрическое поглощение

Лицензионный контент EPCI:

[1] EPCI Эксперты Европейского института пассивных компонентов оригинальные статьи
[2] Справочник по пассивным компонентам CLR от P-O.Фагерхольт *

* используется под авторским правом EPCI от CTI Corporation, США


Содержание этой страницы находится под международной лицензией Creative Commons Attribution-Share Alike 4.0.

Емкость и конденсаторы | Analog Devices

I. Общие сведения о паразитных эффектах в конденсаторах:

Q.Мне нужно понять, как выбрать правильный конденсатор для моего приложения, но я не понимаю достоинств и недостатков множества различных типов.

A. Выбор подходящего типа конденсатора для конкретного применения не так уж и сложен. Как правило, вы обнаружите, что большинство конденсаторов относятся к одной из четырех категорий применения:

  • Соединение по переменному току , включая шунтирование (пропускание сигналов переменного тока при блокировке постоянного тока)
  • развязка (фильтрация переменного тока или высоких частот, наложенных на постоянный или низкие частоты в силовых, опорных и сигнальных цепях)
  • активные / пассивные фильтры RC или частотно-избирательные сети
  • аналоговые интеграторы и схемы выборки и хранения (получение и сохранение заряда)

Несмотря на то, что существует более дюжины или около того популярных типов конденсаторов, включая полимерные, пленочные, керамические, электролитические и т. Д.- вы обнаружите, что, как правило, только один или два типа лучше всего подходят для конкретного применения, потому что явные недостатки или «паразитные эффекты» на производительность системы, связанные с другими типами конденсаторов, заставят их устранить.


В. О каких «паразитических эффектах» вы говорите?

A. В отличие от «идеального» конденсатора, «настоящий» конденсатор характеризуется дополнительными «паразитными» или «неидеальными» компонентами или поведением в виде резистивных и индуктивных элементов, нелинейности и диэлектрической памяти.Результирующие характеристики этих компонентов обычно указываются в паспорте производителя конденсатора. Понимание влияния этих паразитных факторов в каждом приложении поможет вам выбрать правильный тип конденсатора.


В. Хорошо, каковы наиболее важные параметры, описывающие неидеальное поведение конденсатора?

A. Четыре наиболее распространенных эффекта: утечка (параллельное сопротивление), эквивалентное последовательное сопротивление (ESR), эквивалентное последовательное сопротивление (ESL) и диэлектрическое поглощение (память).

Утечка конденсатора, R P : Утечка — важный параметр в приложениях связи по переменному току, в устройствах хранения, таких как аналоговые интеграторы и держатели образцов, а также при использовании конденсаторов в цепях с высоким импедансом.

В идеальном конденсаторе заряд Q изменяется только в зависимости от внешнего тока. Однако в реальном конденсаторе сопротивление утечки позволяет заряду стекать со скоростью, определяемой постоянной времени R-C.

Конденсаторы электролитического типа (тантал и алюминий), отличающиеся высокой емкостью, имеют очень высокий ток утечки (обычно порядка 5-20 нА на мкФ) из-за низкого сопротивления изоляции и не подходят для хранения или связи Приложения.

Наилучшим выбором для соединений и / или хранения является тефлон (политетрафторэтилен) и другие «поли» типы (полипропилен, полистирол и т. Д.).

Эквивалентное последовательное сопротивление (ESR), R S : Эквивалентное последовательное сопротивление (ESR) конденсатора — это сопротивление выводов конденсатора, соединенных последовательно с эквивалентным сопротивлением пластин конденсатора.ESR заставляет конденсатор рассеивать мощность (и, следовательно, производить потери), когда протекают высокие переменные токи. Это может иметь серьезные последствия для ВЧ и разделительных конденсаторов питания, несущих большие токи пульсации, но вряд ли окажет большое влияние на прецизионные низкоомные аналоговые схемы с высоким импедансом.

Конденсаторы с самым низким ESR включают как слюдяные, так и пленочные типы.

Эквивалентная последовательная индуктивность (ESL) , L S : Эквивалентная последовательная индуктивность (ESL) конденсатора моделирует индуктивность выводов конденсатора, соединенных последовательно с эквивалентной индуктивностью пластин конденсатора.Как и ESR, ESL также может быть серьезной проблемой на высоких (RF) частотах, даже если сама прецизионная схема может работать на постоянном токе или на низких частотах. Причина в том, что транзисторы, используемые в прецизионных аналоговых схемах, могут иметь усиление, достигающее переходных частот ( F t ) в сотни МГц или даже несколько ГГц, и могут усиливать резонансы с низкими значениями индуктивности. Это делает важным, чтобы клеммы источника питания таких цепей были должным образом развязаны на высокой частоте.

Электролитические, бумажные или пластмассовые пленочные конденсаторы — плохой выбор для развязки на высоких частотах; в основном они состоят из двух листов металлической фольги, разделенных листами пластика или бумажного диэлектрика и скрученных в рулон. Такая структура имеет значительную самоиндукцию и действует больше как индуктор, чем конденсатор на частотах, превышающих всего несколько МГц.

Более подходящим выбором для ВЧ развязки является монолитный керамический конденсатор с очень низкой последовательной индуктивностью.Он состоит из многослойного сэндвича из металлических пленок и керамического диэлектрика, причем пленки соединены параллельно шинам, а не скручены последовательно.

Незначительный компромисс заключается в том, что монолитные керамические конденсаторы могут быть микрофонными (то есть чувствительными к вибрации), а некоторые типы могут даже быть саморезонансными со сравнительно высокой добротностью из-за низкого последовательного сопротивления, сопровождающего их низкую индуктивность. С другой стороны, дисковые керамические конденсаторы иногда бывают довольно индуктивными, хотя и менее дорогими.


В. Я видел термин «коэффициент рассеяния» в таблицах выбора конденсаторов. Что это?

A. Хороший вопрос. Поскольку утечку, ESR и ESL почти всегда сложно определить по отдельности, многие производители объединяют утечку, ESR и ESL в единую спецификацию, известную как коэффициент рассеяния или DF, который в основном описывает неэффективность конденсатора. DF определяется как отношение энергии, рассеиваемой за цикл, к энергии, запасенной за цикл.На практике это равно коэффициенту мощности диэлектрика или косинусу фазового угла. Если рассеяние на высоких частотах в основном моделируется как последовательное сопротивление, на интересующей критической частоте отношение эквивалентного последовательного сопротивления, ESR, к общему емкостному реактивному сопротивлению является хорошей оценкой DF,

Коэффициент рассеяния также оказывается эквивалентным обратной величине добротности конденсатора, или Q, которая также иногда указывается в технических характеристиках производителя.

Диэлектрическая абсорбция, RDA, CDA: Монолитные керамические конденсаторы отлично подходят для ВЧ развязки, но они имеют значительное диэлектрическое поглощение , что делает их непригодными для использования в качестве запоминающего конденсатора усилителя удержания образца (SHA). Диэлектрическая абсорбция — это гистерезисное внутреннее распределение заряда, которое заставляет конденсатор, который быстро разряжается, а затем размыкается, чтобы восстановить часть своего заряда. Поскольку количество восстановленного заряда является функцией его предыдущего заряда, это, по сути, зарядная память и вызовет ошибки в любом SHA, где такой конденсатор используется в качестве запоминающего конденсатора.

Конденсаторы, рекомендуемые для этого типа применения, включают конденсаторы «поли» типа, о которых мы говорили ранее, то есть полистирол, полипропилен или тефлон. Эти типы конденсаторов имеют очень низкое диэлектрическое поглощение (обычно <0,01%).

Общие характеристики конденсаторов приведены в сравнительной таблице конденсаторов внизу этой страницы.

Замечание о высокочастотной развязке в целом: Лучший способ гарантировать, что аналоговая цепь должным образом развязана как на высоких, так и на низких частотах, — это использовать конденсатор электролитического типа, такой как танталовый шарик, параллельно с монолитно-керамический.Комбинация будет иметь высокую емкость на низкой частоте и останется емкостной до довольно высоких частот. Обычно нет необходимости иметь танталовый конденсатор на каждой отдельной ИС, за исключением критических случаев; Если между каждой ИС и танталовым конденсатором имеется менее 10 см достаточно широкой дорожки для ПК, можно использовать один танталовый конденсатор для нескольких ИС.

Еще одна вещь, о которой следует помнить о высокочастотной развязке, — это фактическое физическое размещение конденсатора.Даже короткие отрезки провода имеют значительную индуктивность, поэтому устанавливайте ВЧ развязывающие конденсаторы как можно ближе к ИС и убедитесь, что выводы состоят из коротких широких дорожек ПК.

В идеале, ВЧ развязывающие конденсаторы должны быть частями для поверхностного монтажа, чтобы исключить индуктивность выводов, но конденсаторы с проволочным концом в порядке, при условии, что длина выводов устройства не превышает 1,5 мм.


II. Паразитная емкость:

Теперь, когда мы поговорили о паразитных эффектах конденсаторов как компонентов, давайте поговорим о другой форме паразитной емкости, известной как «паразитная» емкость.

В. Что это?

A. Что ж, точно так же, как конденсатор с параллельными пластинами, паразитные конденсаторы образуются всякий раз, когда два проводника находятся в непосредственной близости друг от друга (особенно если они идут параллельно), и не закорочены вместе или экранированы проводником, служащим в качестве щит Фарадея.

Паразитная или паразитная емкость обычно возникает между параллельными дорожками на печатной плате или между дорожками / плоскостями на противоположных сторонах печатной платы. Возникновение и влияние паразитной емкости, особенно на очень высоких частотах, к сожалению, часто упускается из виду при моделировании схемы и может привести к серьезным проблемам с производительностью, когда системная печатная плата построена и собрана; примеры включают больший шум, пониженную частотную характеристику, даже нестабильность.

Например, если формула емкости применяется к случаю следов на противоположных сторонах платы, то для материала печатной платы общего назначения (E R = 4,7, d = 1,5 мм) емкость между проводниками на противоположных сторонах платы плата чуть ниже 3 пФ / см 2 . На частоте 250 МГц 3 пФ соответствуют реактивному сопротивлению 212,2 Ом!


В. Итак, как я могу устранить паразитную емкость?

A. Вы никогда не сможете «устранить» паразитную емкость; Лучшее, что вы можете сделать, это принять меры для минимизации его воздействия в цепи.

В. Как мне это сделать?

A. Что ж, один из способов минимизировать эффекты паразитной связи — это использовать экран Фарадея, который представляет собой просто заземленный проводник между источником связи и цепью, на которую воздействуют.

В. Как это работает?

A. Посмотрите на рисунок; это эквивалентная схема, показывающая, как источник высокочастотного шума, V N , связан с полным сопротивлением системы Z через паразитную емкость C. Если у нас мало или совсем нет контроля над V n или местоположением из Z 1 , следующим лучшим решением является установка щита Фарадея:

Как показано ниже, экран Фарадея прерывает электрическое поле связи.Обратите внимание, как экран заставляет шум и токи связи возвращаться к своему источнику, не протекая через Z 1 .

Другой пример емкостной связи — керамические ИС с пайкой сбоку. Эти DIP-пакеты имеют небольшую квадратную проводящую крышку из ковара, припаянную к металлизированному краю на керамической верхней части корпуса. Производители упаковки предлагают только два варианта: металлизированный ободок можно соединить с одним из угловых штырей упаковки или оставить неподключенным. Большинство логических схем имеют вывод заземления в одном из углов корпуса, поэтому крышка заземлена.Но у многих аналоговых схем нет контакта заземления в углу корпуса, и крышка остается плавающей. Такие схемы оказываются гораздо более уязвимыми к шуму электрического поля, чем тот же чип в пластиковом корпусе DIP, где чип неэкранирован.

Каким бы ни был уровень шума окружающей среды, рекомендуется заземлять крышку любой боковой паяной керамической ИС, если крышка не заземлена производителем. Это можно сделать с помощью проволоки, припаянной к крышке (это не повредит устройство, так как микросхема термически и электрически изолирована от крышки).Если пайка к крышке недопустима, можно использовать заземленный зажим из фосфористой бронзы для заземления или использовать токопроводящую краску для соединения крышки с контактом заземления. Никогда не пытайтесь заземлить такую ​​крышку, не убедившись, что она действительно не подключена. ; существуют типы устройств, крышка которых соединена с шиной питания, а не с землей!

Один случай, когда экран Фарадея неосуществим, — это между соединительными проводами интегральной микросхемы. Это имеет важные последствия.Паразитная емкость между двумя соединительными проводами микросхемы и связанными с ними выводами составляет порядка 0,2 пФ; наблюдаемые значения обычно лежат между 0,05 и 0,6 пФ.

Рассмотрим преобразователь высокого разрешения (АЦП или ЦАП), который подключен к высокоскоростной шине данных. Каждая линия шины данных (которая будет переключаться со скоростью от 2 до 5 В / нс) сможет влиять на аналоговый порт преобразователя через эту паразитную емкость; последующее объединение цифровых фронтов ухудшит характеристики преобразователя.

Этой проблемы можно избежать, изолировав шину данных, вставив заблокированный буфер в качестве интерфейса. Хотя это решение включает в себя дополнительный компонент, который занимает площадь на плате, потребляет электроэнергию и увеличивает стоимость, оно может значительно улучшить отношение сигнал-шум преобразователя.


ТИП ТИПИЧНЫЙ
ДИЭЛЕКТРИЧЕСКИЙ
ПОГЛОЩЕНИЕ
ПРЕИМУЩЕСТВА НЕДОСТАТКИ
НПО керамика <0.1% Маленький размер корпуса
Недорого
Хорошая стабильность
Широкий диапазон значений
Многие производители
Низкая индуктивность
DA обычно низкий, но не может быть указан
Ограничено небольшими значениями (10 нФ)
Полистирол 0,001%
до 0,02%
Недорого
Доступен низкий DA
Широкий диапазон значений
Хорошая стабильность
Повреждено температурой> + 85 ° C
Большой размер корпуса
Высокая индуктивность
Полипропилен 0.001%
до 0,02%
Недорого
Доступен низкий DA
Широкий диапазон значений
Повреждено температурой> + 105 ° C
Большой размер корпуса
Высокая индуктивность
тефлон 0,003%
до 0,02%
Доступен низкий DA
Хорошая стабильность
Эксплуатация при температуре выше + 125 ° C
Широкий диапазон значений
Относительно дорого
Большой размер
Высокая индуктивность
МОП 0.01% Good DA
Small
Эксплуатация при температуре выше + 125 ° C
Низкая индуктивность
Ограниченная доступность
Доступна только для малых значений емкости
Поликарбонат 0,1% Хорошая стабильность
Низкая стоимость
Широкий диапазон температур
Большой размер
DA ограничивает 8-битные приложения
Высокая индуктивность
Полиэстер 0.От 3%
до 0,5%
Средняя стабильность
Низкая стоимость
Широкий температурный диапазон
Низкая индуктивность (многослойная пленка)
Большой размер
DA ограничивает 8-битные приложения
Высокая индуктивность
Монолитная керамика
(High K)
> 0,2% Низкая индуктивность
Широкий диапазон значений
Плохая стабильность
Плохая DA
Высокий коэффициент напряжения
Слюда> 0.003% Низкие потери на ВЧ
Низкая индуктивность
Очень стабильная
Доступны значения 1% или лучше
Довольно большой
Низкие значения (<10 нФ)
Дорого
Алюминий электролитический Высокая Большие значения
Высокие токи
Высокие напряжения
Малые размеры
Высокая утечка
Обычно поляризация
Низкая стабильность
Низкая точность
Индуктивная
Тантал электролитический Высокая Малый размер
Большие значения
Средняя индуктивность
Довольно высокая утечка
Обычно поляризованный
Дорого
Плохая стабильность
Низкая точность

Конденсаторы и емкость

Различные типы конденсаторов

Конденсатор — электронное устройство для накопления заряда.Конденсаторы можно найти почти во всех электронных схемах, кроме самых простых. Есть много разных типов конденсаторов, но все они работают одинаково. Упрощенный вид конденсатора представляет собой пару металлических пластин, разделенных зазором, в котором находится изолирующий материал, известный как диэлектрик. Этот упрощенный конденсатор также выбран, поскольку символ электронной схемы для конденсатора представляет собой пару параллельных пластин, как показано на рисунке 1.

Рис. 1. Символ неполяризованного конденсатора.

Обычно электроны не могут войти в проводник, если не существует пути для выхода равного количества электронов. Однако лишние электроны могут быть «втиснуты» в проводник без пути для выхода, если электрическое поле может развиваться в пространстве относительно другого проводника. Количество дополнительных свободных электронов, добавленных к проводнику (или отнятых свободных электронов), прямо пропорционально величине потока поля между двумя проводниками.

В этом упрощенном конденсаторе диэлектрик — воздух.Когда на клеммы конденсатора подается напряжение В, , электроны перетекают на одну из пластин и уносятся с другой пластины. Общее количество электронов в конденсаторе остается прежним. На одной отрицательной пластине их больше, а на положительной — меньше.

Рисунок 2. Зарядка конденсатора от аккумулятора

Если бы напряжение было увеличено, увеличившаяся разность потенциалов между пластинами подтолкнула бы больше электронов к отрицательно заряженной пластине.Мы могли измерить заряд, накопленный на пластине, в зависимости от различных приложенных напряжений.

При нулевом напряжении пластины конденсатора нейтральны, поэтому заряд не сохраняется. (мы предполагаем, что мы начали с полностью разряженного конденсатора), при напряжении V заряд на пластинах равен Q, а при удвоенном напряжении заряд удваивается. Мы обнаруживаем, что с увеличением напряжения заряд увеличивается линейно. Мы можем построить это как прямую линию.

Предположим, что мы уходим, проводим некоторые исследования и возвращаемся с более совершенным конденсатором, который хранит больше заряда для данного напряжения, мы можем построить график зависимости сохраненного заряда от приложенного напряжения.

Это можно было бы представить как другую линию с более крутым уклоном.Если бы мы построили много графиков для разных конденсаторов, мы получили бы много прямых линий. Мы можем сказать, что мера емкости — это то, сколько заряда сохраняется при заданном напряжении. Иногда это выражается как Q = CV .

Конечно, при зарядке конденсатора должна выполняться работа по перемещению заряда. Следовательно, необходимо подавать энергию, и эта энергия доступна, когда конденсатор разряжен.

Проделанная работа определяется как W = qV .Первоначально заряд легко перемещается на пластины конденсатора, однако по мере того, как больше заряда перемещается на пластины конденсатора, сила отталкивания между зарядами затрудняет добавление заряда, когда сила отталкивания зарядов равна мощности батареи, больше нельзя перемещать заряд на пластины. Следовательно, средняя работа составляет 1/2 qV . Если мы посмотрим на наш график зависимости заряда от напряжения, то увидим, что это то же самое, что и площадь под кривой. В общем, проделанная работа равна переданной энергии.Математически,

Факторы, влияющие на емкость

Как увеличить емкость конденсатора с параллельными пластинами? На емкость конденсатора с параллельными пластинами влияют три фактора.

Площадь

Конденсатор переменной емкости

Увеличивая площадь пластин, мы можем поместить на пластины больше заряда, прежде чем силы отталкивания станут проблемой. Следовательно, емкость пропорциональна площади перекрытия пластин.В переменном конденсаторе площадь перекрытия может быть увеличена или уменьшена путем вращения взаимопроникающих пластин, таким образом увеличивая или уменьшая емкость. Пластины электролитических конденсаторов протравлены для получения шероховатой поверхности, которая еще больше увеличивает площадь поверхности.

Разделение

Уменьшение расстояния между пластинами снижает напряжение конденсатора, поскольку электрическое поле не зависит от расстояния между пластинами. Напряжение на конденсаторе В = Ед.Следовательно, напряжение увеличивается. Для постоянного заряда Q , C = Q / V = ​​Q / Ed.

Диэлектрическая проницаемость

Емкость конденсатора с параллельными пластинами определяется выражением C = ε r A / d , где A — площадь пластин, d — расстояние между пластинами, а ε r — площадь пластин. относительная проницаемость диэлектрика между пластинами. Относительная проницаемость — это некоторый коэффициент, K , умножающий допустимость свободного пространства ε 0 0 имеет значение 8,85×10 -12 F.m -1 .

Полный список относительных допусков можно найти практически для любого диэлектрического материала. Чем больше относительная проницаемость, тем больше емкость конденсатора. Хорошие материалы — слюда, полистирол, масло.

ε r = K ε 0

Конденсаторные сети

Рисунок 3. Последовательные и параллельные сети конденсаторов

серии

Рассмотрим последовательную сеть конденсаторов, показанную на рисунке 3a.где положительная пластина соединена с отрицательной пластиной следующего. Какова эквивалентная емкость сети? Посмотрите на пластины посередине, эти пластины физически отключены от цепи, поэтому общий заряд на них должен оставаться постоянным. Следовательно, когда на оба конденсатора подается напряжение, заряд + Q на положительной пластине конденсатора C 1 должен уравновешиваться зарядом — Q на отрицательной пластине конденсатора C. 2 .В результате оба конденсатора обладают одинаковым зарядом Q. Падения потенциала В 1 и В 2 на двух конденсаторах, как правило, различаются. Однако сумма этих падений равна общему падению потенциала В , приложенному к входным и выходным проводам. В = В 1 + В 2 . Эквивалентная емкость пары снова составляет C T = Q / V .Таким образом, 1/ C T = V / Q = ( V 1 + V 2 ) / Q = V 1 / Q V 2 / Q дающий

Как правило, для конденсаторов N , соединенных последовательно, это

Соединяя конденсаторы в сериях, вы сохраняете меньше заряда, так есть ли смысл подключать конденсаторы последовательно? Иногда это делается потому, что конденсаторы имеют максимальное рабочее напряжение, и, если последовательно соединить два конденсатора с максимальным напряжением 900 вольт, вы можете увеличить рабочее напряжение до 1800 вольт.

Параллельный

Для параллельной схемы, такой как на рисунке 3b. напряжения одинаковы для всех компонентов. Однако общий заряд делится между двумя конденсаторами, поскольку он должен распределяться таким образом, чтобы напряжение на них было одинаковым. Кроме того, поскольку конденсаторы могут иметь разные емкости C 1 и C 2 , заряды Q 1 и Q 2 также должны быть разными.Эквивалентная емкость C T пары конденсаторов — это просто отношение Q / V , где Q = Q 1 + Q 2 — это общий накопленный заряд. Отсюда следует, что C T = Q / V = (Q 1 + Q 2 ) / V = Q 1 / V + Q 2 / В подача

Из предыдущего обсуждения довольно очевидно, что для конденсаторов Н , подключенных параллельно, общая емкость составляет

Общая емкость увеличивается за счет параллельного добавления конденсаторов, поэтому мы получаем большую емкость, чем это возможно при использовании одного конденсатора.В лабораториях физики высоких энергий часто есть большие батареи конденсаторов, которые могут хранить большое количество энергии, которая высвобождается за очень короткое время. Самая большая батарея конденсаторов в 2006 году может хранить 50 МДж энергии.

Конденсаторы зарядные и разрядные

Цепь, состоящая из батареи, переключателя, резистора и конденсатора в последовательном контуре, называется RC-цепью. Закон Кирхгофа по напряжению для этой схемы имеет вид

В = ИК + Q / С .Если выразить чисто в терминах заряда, это становится

V = dQ / dt R + Q / C .

Это дифференциальное уравнение, решение которого является экспоненциальной функцией. Когда переключатель замкнут, конденсатор со временем заряжается:

Q = Q f (1 — e -t / RC ),

, где Q — это заряд в момент времени t , а Q f — последний заряд конденсатора.Обратите внимание, что Q никогда не равно Q f , но поскольку t становится чрезвычайно большим, Q становится произвольно близким к Q f . Произведение RC называется постоянной времени RC и является характеристической величиной RC цепи. Когда t = RC , конденсатор заряжен до доли (1 — 1/ e , около 63%) от своего окончательного значения. Необходимо использовать постоянную времени, а не какое-то конечное время, поскольку процесс асимптотический.Его значение — произвольный выбор; мы, естественно, выбираем значение в терминах экспоненциального основания (когда показатель отрицательный).

Мигающая анимация 1. Измените значения резистора и конденсатора, чтобы увидеть влияние на время достижения конденсатором пикового напряжения.

Flash Animation 2. Зарядите конденсатор, пока он не достигнет пика, а затем разрядите его. Как значения R и C влияют на процессы?

Типы конденсаторов

Конденсаторы электролитические

Алюминиевые электролитические конденсаторы изготавливаются путем наложения электролитической бумаги между анодной и катодной фольгами и последующего наматывания результата.Процесс изготовления электрода, обращенного к поверхности протравленной анодной фольги, чрезвычайно сложен. Следовательно, противоположный электрод создается путем заполнения конструкции электролитом. Благодаря этому процессу электролит по существу выполняет роль катода.

Электролитические конденсаторы пропитаны жидкостью или бумагой, пропитанной жидкостью, которая не является диэлектриком, но при приложении напряжения создает слой оксида алюминия, который действует как диэлектрик. Реакция зависит от полярности приложенного напряжения.Если полярность поменять, конденсатор будет выделять газ и, вероятно, взорвется или лопнет из-за давления внутри, поэтому он не подходит для применения с переменным током.

MEMs Конденсаторы

Микро-электромеханические системы (MEM) — это небольшие устройства, изготовленные из кремния. Могут быть изготовлены пластинчатые конденсаторы с небольшими изменениями емкости при увеличении или уменьшении расстояния между пластинами. Небольшие устройства можно использовать как датчики и гироскопы.

Распространенными типами устройств являются конденсаторы с параллельными пластинами для определения положения. Кроме того, взаимопроникающие гребенчатые конструкции, в которых емкость может быть изменена с использованием перемещения одной гребенки относительно другой, либо в поперечном направлении, либо в продольном направлении. Из-за их небольшого размера изменение емкости очень мало, порядка 10 -15 Ф (фемто-Фарад).

Конденсаторы танталовые

Танталовые конденсаторы поляризованы и имеют низкое напряжение, как электролитические конденсаторы.Они дороги, но очень малы, поэтому используются там, где требуется большая емкость в небольших размерах, например, в мобильных телефонах или портативных компьютерах. Эти конденсаторы становятся все более важными, поскольку растет спрос на все меньшие электронные устройства. Колумбит-танталит — колтан , сокращенно руда, из которой очищается тантал, добывается в Австралии, г. Егпыт. Высокий спрос на руду также финансировал гражданские войны в Демократической Республике Конго. В докладе Совета безопасности ООН утверждается, что большая часть руды добывается незаконно и переправляется через восточные границы страны вооруженными формированиями из соседних Уганды, Бурунди и Руанды, обеспечивая доход для финансирования военной оккупации Конго.

Суперконденсаторы

Суперконденсаторы — это конденсаторы, которые обладают способностью накапливать большие количества заряда и, следовательно, энергии в очень небольшом объеме. Накопление энергии происходит за счет статического заряда, а не электрохимического процесса, присущего батарее. Применение разности напряжений на положительной и отрицательной пластинах заряжает суперконденсатор. Эта концепция похожа на электрический заряд, который накапливается при ходьбе по ковру. Впервые суперконденсатор был задуман в 1957 году, но теперь исследования сосредоточены на их использовании в качестве источников питания с малым весом в качестве альтернативы батареям.суперконденсатор переходит в аккумуляторную технологию благодаря использованию специальных электродов и небольшого количества электролита. Суперконденсаторы могут найти применение в таких случаях, как временные резервные источники питания в электросети или обеспечение начального всплеска энергии для движения электромобилей.

Сводка

Конденсаторы

обладают способностью заряжать и высвобождать накопленный заряд очень быстро, что позволяет им функционировать разными способами. Они занимают важное место во всем: от схем стабилизации напряжения в чувствительной электронике до помощи в преобразовании переменного тока в постоянный для зарядки аккумуляторов во всем, от мобильных скутеров до портативных компьютеров.

Конденсаторы — это устройства, накапливающие заряд. Емкость определяется как отношение накопленного заряда к единице напряжения. C = Q / V

Емкость конденсатора с параллельными пластинами определяется как C = ε r A / d .

Энергия, запасенная в конденсаторе, рассчитывается по работе, совершаемой при перемещении заряда по пластинам. dW = V dq . Накопитель энергии — это область под графиком заряда / напряжения. 1 / 2QV или из C = Q / V, 1 / 2CV 2 = 2Q 2 / C.

ЕМКОСТЬ

ЕМКОСТЬ ЕМКОСТЬ

Еще одно важное свойство цепей переменного тока, помимо сопротивления и индуктивности, это емкость. В то время как индуктивность представлена ​​в цепи катушкой, емкость представлена ​​конденсатором.

Любые два проводника, разделенные непроводником, называемым диэлектриком, составляют конденсатор. В электрической цепи конденсатор выполняет роль резервуар или хранилище для электричества.

Когда конденсатор подключен к источнику прямого ток, такой как аккумуляторная батарея в схеме, показанной на рисунке 8-176, и переключатель затем замыкается, пластина с маркировкой B становится положительно заряженной, и пластина А заряжена отрицательно. Ток течет во внешней цепи в течение времени электроны движутся от B к A. Текущий поток в цепи максимальна в момент замыкания переключателя, но постоянно затем уменьшается, пока не достигнет нуля.Ток становится равным нулю, когда как только разница в напряжении A и B станет такой же, как напряжение батареи. Если выключатель разомкнут, пластины остаются заряженными. Тем не мение, конденсатор быстро разряжается при коротком замыкании.

Количество электричества, которое может хранить конденсатор, зависит от нескольких факторов, включая тип материала диэлектрика. Прямо пропорционально к площади пластины и обратно пропорционально расстоянию между тарелки.

На рисунке 8-177 размещены две плоские металлические пластины. близко друг к другу (но не касаясь). Обычно пластины электрически нейтральный; то есть ни на одной из пластин не будет электрического заряда. В момент замыкания переключателя в положение батареи счетчик покажет определенный скачок тока в одном направлении, но почти мгновенно вернется к нулю.

Если затем вынуть аккумулятор из цепи и выключатель замкнется в положении конденсатора счетчик снова показывает кратковременный скачок тока, но на этот раз в противоположном направлении.Из этого эксперимента очевидно что две пластины накапливают энергию при подключении к источнику напряжения, и высвобождает энергию при коротком замыкании. Две пластины составляют простой электрический конденсатор, или конденсатор, и обладают свойством хранения электричество. На самом деле энергия хранится в электрическом или диэлектрическом, поле между пластинами.

Также должно быть ясно, что во время работы конденсатора заряжен или разряжен, в цепи есть ток, даже если цепь разорвана зазором между пластинами конденсатора.Однако там ток только во время заряда и разряда, и в этот период времени очень мало. Не может быть постоянного движения постоянного тока через конденсатор. Хороший конденсатор блокирует постоянный ток (не пульсирует постоянного тока) и пропускает воздействие переменного тока.

Заряд электричества, который может быть помещен на конденсатор, пропорционален приложенному напряжению и емкости конденсатора (конденсатора). Емкость зависит от общей площади пластин, толщины диэлектрик и состав диэлектрика.

Если тонкий лист бакелита (заполненный слюдой) заменить воздух между обкладки конденсатора, например, емкость будет увеличена примерно пять раз.

Любой электрический заряд, произведенный приложенным напряжением и ограниченный изолятор (диэлектрик) создает диэлектрическое поле. Как только поле создан, он имеет тенденцию противодействовать любому изменению напряжения, которое может повлиять на его исходный должность. Все цепи содержат некоторую емкость, но если они не содержат единица, называемая конденсатором, емкость, для всех практических целей, игнорируется.Два проводника, называемые электродами или пластинами, разделены из непроводника (диэлектрика) составляют простой конденсатор. Пластины могут быть изготовленным из меди, олова или алюминия. Часто их делают из фольги. (металлы, спрессованные в тонкие листы и пригодные для прокатки). Диэлектрик может быть воздух, стекло, слюда или электролит, состоящий из оксидной пленки, но используемый тип будет определять величину напряжения, которое может быть приложено, и количество энергии, которое будет сохранено. Диэлектрические материалы имеют различные атомные структуры и представляют различное количество атомов для электростатическое поле.Все диэлектрические материалы сравнивают с вакуумом. и имеют числовое значение в соответствии с соотношением пропускной способности между их. Номер, присвоенный материалу, основан на той же площади и толщине. как используется в вакууме. Числа, используемые для выражения этого отношения, называются диэлектрические постоянные и обозначаются буквой «К.»

Если заменить источник переменного тока батареи, конденсатор действует совершенно иначе, чем с прямым Текущий.Когда в цепь подается переменный ток (рисунок 8-179), заряд на пластинах постоянно меняется. Это означает, что электричество сначала должен течь от Y по часовой стрелке до X, затем от X против часовой стрелки по кругу до Y, затем от Y по часовой стрелке до X и так далее. Хотя нет через изолятор между пластинами конденсатора протекает ток, он постоянно течет в оставшейся части цепи между X и Y. цепь, в которой есть только ёмкость, ток уводит втянутый напряжение по сравнению с цепью, в которой есть индуктивность, где ток отстает от напряжения.

Единицей измерения емкости является фарада, для которой обозначен символ буква «f». Фарад слишком велик для практического использования, и единицы измерения обычно используются микрофарады, одна миллионная фарада и микромикрофарада, одна миллионная микрофарада.

Типы конденсаторов

Конденсаторы можно разделить на две группы: фиксированные и переменные. Фиксированный конденсаторы, которые имеют приблизительно постоянную емкость, тогда могут быть далее делятся, в зависимости от типа используемого диэлектрика, на следующие классы: бумага, масло, слюда, электролитические конденсаторы.Керамические конденсаторы также используются в некоторых схемах.

При подключении электролитических конденсаторов в цепь соблюдайте полярность. необходимо соблюдать. Бумажные конденсаторы могут иметь одну клемму с пометкой «земля», Это означает, что этот вывод подключается к внешней фольге. Полярность делает обычно не требуется при использовании соединительной бумаги, масла, слюды или керамики. конденсаторы.

Бумажные конденсаторы

Пластины бумажных конденсаторов представляют собой полосы металлической фольги, разделенные вощеная бумага (рисунок 8-180).Диапазон емкости бумажных конденсаторов от примерно 200 ммс до нескольких мф. Полоски фольги и бумаги свернуты вместе, чтобы сформировать цилиндрический картридж, который затем запечатывается воском для не допускать попадания влаги и предотвращения коррозии и утечки. Два металлических провода припаяны к пластинам, по одной на каждом конце цилиндра. Сборка заключена либо в картонную обложку, либо в твердый формованный пластиковое покрытие.

Конденсаторы ванны состоят из бумажных картриджей конденсаторов. герметично закрывается в металлических контейнерах.Контейнер часто служит общая клемма для нескольких закрытых конденсаторов, но если это не клемма, крышка служит экраном от электрических помех (рисунок 8-181).
Конденсаторы масляные

В радио- и радиолокационных передатчиках напряжение достаточно высокое, чтобы вызвать искрение, или пробой бумажных диэлектриков. Следовательно, в конденсаторы, в которых используется масло или пропитанная маслом бумага для диэлектрический материал является предпочтительным.Конденсаторы этого типа значительно дороже обычных бумажных конденсаторов, и их использование обычно ограничивается радио- и радиолокационным передающим оборудованием (рисунок 8-182).

Слюдяные конденсаторы

Фиксированный слюдяной конденсатор изготовлен из металлических пластин из фольги, которые разделены между собой. листами слюды, образующими диэлектрик. Вся сборка покрыта из литого пластика, не пропускающего влагу. Слюда — отличный диэлектрик и выдерживает более высокое напряжение, чем бумага, не допуская искрения между тарелки.Общие значения слюдяных конденсаторов колеблются примерно от 50 микрофарад, примерно до 0,02 мкФ. Показаны слюдяные конденсаторы. на рисунке 8-183.

Конденсаторы электролитические

Для емкостей более нескольких микрофарад площади пластин бумажные или слюдяные конденсаторы должны становиться очень большими; таким образом, электролитические конденсаторы обычно используются вместо них. Эти блоки обеспечивают большую емкость в небольшие физические размеры. Их значения колеблются от 1 до примерно 1500 микрофарад.В отличие от других типов, электролитические конденсаторы обычно поляризованы, и должен подвергаться воздействию постоянного напряжения или пульсирующего постоянного напряжения. Только; однако для использования используется специальный тип электролитического конденсатора. в моторах.

Электролитический конденсатор широко используется в электронных схемах и состоит из двух металлических пластин, разделенных электролитом. Электролит в контакте с отрицательной клеммой, в пастообразной или жидкой форме, содержит отрицательный электрод.Диэлектрик очень тонкий. пленка оксида, нанесенная на положительный электрод конденсатора. В положительный электрод, который представляет собой алюминиевый лист, сложен для достижения максимальной область. Конденсатор подвергается процессу формования во время производства, в котором через него проходит ток. Поток тока приводит к нанесение тонкого слоя оксида на алюминиевую пластину.

Близкое расположение отрицательного и положительного электродов приводит к сравнительно высокому значению емкости, но допускает большую вероятность пробоя напряжения и утечки электронов с одного электрода на Другие.

Используются два типа электролитических конденсаторов: (1) мокрый электролитический. и (2) сухие электролитические конденсаторы. В первом случае электролит жидкость и емкость должны быть герметичными. Этот тип всегда должен быть устанавливается в вертикальном положении.

Электролит установки сухого электролиза представляет собой пасту, содержащуюся в разделитель из абсорбирующего материала, такого как марля или бумага. Сепаратор не только удерживает электролит на месте, но и предотвращает короткое замыкание тарелки.Конденсаторы сухие электролитические изготавливаются как цилиндрические, так и прямоугольная форма блока и может содержаться как внутри картона, так и металлические крышки. Поскольку электролит не может пролиться, сухой конденсатор может монтироваться в любом удобном положении. Показаны электролитические конденсаторы. на рисунке 8-184.

Конденсаторы параллельно и последовательно

Конденсаторы могут быть объединены параллельно или последовательно для получения эквивалентных значения, которые могут быть либо суммой отдельных значений (параллельно) или значение меньше наименьшей емкости (последовательно).Фигура 8-185 показаны параллельные и последовательные соединения.

При измерении емкости используются две единицы: фарада и кулон. Как было определено ранее, фарад — это величина емкости. присутствует в конденсаторе, когда хранится один кулон электрической энергии на пластины, и на конденсатор подается один вольт. Один кулон это электрический заряд 6,28 миллиарда миллиардов электронов. Из этого видно, что

В A на рисунке 8-185 напряжение E — это то же самое для всех конденсаторов.Общий заряд Qt представляет собой сумму всех индивидуальные сборы, Q1, Q2 и Q3.

Используя основное уравнение для конденсатора,

Общий заряд равен Qt = CtE, где Ct — общая емкость. С общий заряд конденсаторов, включенных параллельно, является суммой отдельных заряды конденсаторов,

Qt = Q1 + Q2 + Q3.

Используя оба уравнения для полного заряда, получаем уравнение

CtE = C1E + C2E + C3E.

Разделив обе части этого уравнения на E, получим

Ct = C1 + C2 + C3.

Эта формула используется для определения общей емкости любого числа конденсаторов параллельно. При последовательном расположении (B на рис. 8-185) ток одинаков во всех частях цепи. Каждый конденсатор вырабатывает напряжение во время заряда, и сумма напряжений всех конденсаторы должны быть равны приложенному напряжению E. По уравнению конденсатора приложенное напряжение, E, равно общему заряду, деленному на общий емкость, или

Общий заряд Qt равен заряду любого из конденсаторов. потому что один и тот же ток течет у всех в течение одного и того же промежутка времени, и поскольку заряд равен току, умноженному на время в секундах (Qt = I х Т).Следовательно,

Qt = Q1 = Q2 = Q3,

и

, так как в цепи с последовательно включенными конденсаторами

Et = E1 + E2 + E3,

где E1, E2 и E3 — напряжения трех конденсаторов. потом

Разделив обе части уравнения на Qt, получим

Величина, обратная общей емкости любого количества конденсаторов. in series равно сумме обратных величин отдельных значений.

Параллельные конденсаторы объединяются по правилу, аналогичному тому, которое используется для объединения резисторы последовательно. Последовательные конденсаторы объединяются по правилу, аналогичному этому. для комбинирования параллельных резисторов.

При последовательном подключении двух конденсаторов C1 и C2 общая емкость дается уравнением:

Номинальное напряжение конденсаторов

При выборе или замене конденсатора для использования в конкретной цепи, необходимо учитывать следующее: (1) Требуемое значение емкости. и (2) величина напряжения, которому должен подвергаться конденсатор.Если напряжение, приложенное к пластинам, слишком велико, диэлектрик будет сломается и возникнет дуга между пластинами. Конденсатор тогда короткое замыкание, и возможное протекание через него постоянного тока может вызвать повреждение других частей оборудования. Конденсаторы имеют напряжение рейтинг, который нельзя превышать.

Рабочее напряжение конденсатора — это максимальное напряжение, которое может наносить равномерно без опасности перегорания дуги. Рабочее напряжение зависит от от (1) типа материала, используемого в качестве диэлектрика и (2) толщины диэлектрика.

Номинальное напряжение конденсатора является фактором при определении емкости. потому что емкость уменьшается с увеличением толщины диэлектрика. Конденсатор высокого напряжения с толстым диэлектриком должен иметь большую площадь пластины, чтобы иметь такую ​​же емкость, как и у аналогичного низкого напряжения конденсатор с тонким диэлектриком. Сила некоторых часто используемых диэлектрические материалы перечислены на рисунке 8-186. Номинальное напряжение также зависит от частоты, потому что потери и результирующий эффект нагрева, увеличиваются с увеличением частоты.

Конденсатор, который можно безопасно заряжать до 500 В постоянного тока, не может быть безопасно подвергается воздействию переменного или пульсирующего постоянного тока, действующее значение которого составляет 500 вольт. An переменное напряжение 500 вольт (среднеквадратичное значение) имеет пиковое напряжение 707 вольт, а конденсатор, к которому он приложен, должен иметь рабочее напряжение не менее 750 вольт. Конденсатор следует подбирать так, чтобы он работал напряжение, по крайней мере, на 50 процентов больше, чем максимальное подаваемое напряжение к нему.

Емкость Реактивное сопротивление

Емкость, как и индуктивность, противодействует протеканию тока. Это сопротивление называется емкостным реактивным сопротивлением и измеряется в омах. Символ емкостного реактивного сопротивления — Xc. Уравнение,

аналогичен закону Ома и уравнению для тока в индуктивном схема. Чем больше частота, тем меньше реактивное сопротивление. Следовательно емкостное реактивное сопротивление,

Проблема:

Предполагается последовательная цепь, в которой подаваемое напряжение составляет 110 вольт. при 60 гц, а емкость конденсатора 80.Найдите емкостное реактивное сопротивление и ток.

Решение:

Чтобы найти емкостное сопротивление, уравнение заменяется на фарады путем деления 80 на 1000000, поскольку 1 миллион микрофарад равен 1 фараду. Это частное равно 0,000080 фарада. Это заменено в уравнении и


Найдите текущий расход: =

Емкостные реактивные сопротивления последовательно и параллельно

При последовательном соединении конденсаторов полное реактивное сопротивление равно сумме индивидуальных реактивных сопротивлений.Таким образом,

Xct = (Xc) 1 + (Xc) 2

Суммарное реактивное сопротивление конденсаторов, подключенных параллельно, находится в таким же образом вычисляется полное сопротивление в параллельной схеме:

Фазы тока и напряжения в реактивных цепях

Когда ток и напряжение проходят через ноль и достигают максимального значения при в то же время говорят, что ток и напряжение находятся в фазе (A на рисунке 8-187).Если ток и напряжение проходят через ноль и достигают максимальные значения в разное время, ток и напряжение считаются равными быть не в фазе. В цепи, содержащей только индуктивность, ток достигает максимальное значение позже напряжения, отставание от напряжения на 90 °, или четверть цикла (B на рисунке 8-187). В цепи, содержащей только емкость, ток достигает максимального значения перед напряжением и током опережает напряжение на 90 °, или на одну четверть цикла (C на рисунке 8-187).Насколько ток отстает или опережает напряжение в цепи, зависит от относительные величины сопротивления, индуктивности и емкости в схема.

Емкость и конденсатор

Устройство, обеспечивающее чистую емкость, называется конденсатором. Конденсатор — это двухконтактный накопитель энергии, который накапливает энергию в своем электрическом поле. Встречается в цепях поворота и фильтрации.

Он состоит из двух металлических пластин, обращенных друг к другу и разделенных диэлектрической средой, такой как воздух, керамика, слюда или бумага.Эти пластины могут нести в себе электрические заряды.

Имея две пластины очень близко друг к другу и разделенные диэлектрической средой, можно собирать большое количество заряда на пластинах. Когда на пластины подается напряжение, источник создает положительный заряд на одной пластине и отрицательный заряд на другой пластине. Заряды видят противоположный заряд на противоположной пластине и притягиваются к нему, а не противостоят аналогичным зарядам на той же пластине.

Определение емкость

Емкость можно определить как свойство конденсатора, которое позволяет конденсатору накапливать заряды на своих проводящих пластинах.Это зависит от физических размеров пластин и проницаемости разделяющего их диэлектрика.

Емкость конденсатора изменяется, как показано ниже:

  • Чем больше площадь пластин, тем выше будет емкость.
  • Больше расстояние разделения между пластин, меньше будет емкость.
  • Повышенная проницаемость разделительной средний, больше будет емкость.

Количество заряда q, хранящегося в конденсаторе, когда напряжение V, приложенный к клеммам конденсатора, равен

.

Q = CV

Где C — емкость конденсатора, а это отношение заряда одной пластины конденсатора к напряжению, приложенному между тарелки.

Единица емкости

Единица измерения емкости — фарад (Ф). Один фарад можно определить как меру емкости конденсатора, который может накапливать один кулон заряда на своих пластинах, когда к нему приложена разность потенциалов в один вольт. Единица конденсатора была названа в честь Мишеля Фарадея за открытие электромагнитной индукции, на основе которой работают двигатели и генераторы.

Соотношение напряжение-ток в конденсаторе

Как мы знаем, ток — это поток электронов.Дифференцируя приведенное выше уравнение ( Q = CV ) по времени t,

dQ / dt = C. dV / dt

dQ / dt — это скорость изменения сборов, которая есть не что иное, как Текущий. Следовательно,

Ток i = C. dV / dt

Следовательно, ток, протекающий через конденсатор, пропорционален скорости изменения напряжения на выводах конденсатора, и конденсатор предотвращает быстрое изменение этого напряжения. Но ток может течь мгновенно. Вот почему резистор предварительной зарядки используется перед конденсатором.Идеальный конденсатор не рассеивает энергию, а накапливает ее и полностью доставляет в схему.

На коммерческом рынке доступны различные типы конденсаторов. Самыми основными категориями являются конденсатор постоянной емкости, поляризованный конденсатор и конденсатор переменной емкости. Клеммы конденсатора постоянной емкости взаимозаменяемы. У поляризованных конденсаторов заранее определены анод и катод. При замене они могут привести к взрыву.

Конденсатор действует как разомкнутый переключатель в цепи постоянного тока.Когда на конденсатор подается постоянное напряжение, на обеих пластинах на короткое время накапливаются заряды. После накопления зарядов через него не проходит ток, и он действует как разомкнутый выключатель.

Емкость

  • • Что такое емкость?
  • • Диэлектрик.
  • • диэлектрическая проницаемость.
  • • Диэлектрическая прочность и максимальное рабочее напряжение.
  • • Расчет заряда конденсатора.

Емкость

Количество энергии, которое может хранить конденсатор, зависит от величины или ЕМКОСТИ конденсатора. Емкость (символ C) измеряется в базовой единице FARAD (символ F). Один фарад — это величина емкости, которая может хранить 1 кулон (6,24 x 10 18 электронов), когда он заряжен до напряжения 1 вольт. Однако Фарада слишком большая единица для использования в электронике, поэтому следующие единицы емкости более полезны.

Дополнительный блок Аббревиатура Стандартное обозначение
мкФ мкФ х 10 -6
нано Фарады нФ х 10 -9
пик Фарад пФ х 10 -12

Однако помните, что при решении задач, связанных с емкостью, формулы и используемые значения должны быть в основных единицах измерения: фарадах, вольтах и ​​т. Д.Поэтому при вводе значения 0,47 нФ, например, в формулу (или ваш калькулятор), его следует вводить в фарадах, используя версию стандартной формы для инженерных обозначений: 0,47 x 10 -9 (Загрузите буклет «Советы по математике», чтобы узнать больше Информация).

Емкость зависит от четырех вещей;

1. Площадь плит

2. Расстояние между пластинами

3. Тип диэлектрического материала

4. Температура

Из этих четырех наименьшее влияние на большинство конденсаторов оказывает температура.Стоимость большинства конденсаторов довольно стабильна в «нормальном» диапазоне температур.

Значения конденсатора могут быть фиксированными или переменными. Большинство переменных конденсаторов имеют очень маленькое значение (несколько десятков или сотен пФ). Значение может быть изменено на:

.
  • • Изменение площади пластин.
  • • Изменение толщины диэлектрика.

Емкость (C) ПРЯМО ПРОПОРЦИОНАЛЬНО ПЛОЩАДИ ДВУХ ПЛАСТИН , которые непосредственно перекрываются, чем больше площадь перекрытия, тем больше емкость.

Емкость ОБРАТНО ПРОПОРЦИОНАЛЬНО РАССТОЯНИЮ МЕЖДУ ПЛАСТИНАМИ. т.е. при раздвигании пластин емкость уменьшается.

Диэлектрик

Электроны на одной пластине конденсатора влияют на электроны на другой пластине, вызывая искажение орбит электронов внутри диэлектрического материала (изолирующего слоя между пластинами). Величина искажения зависит от природы диэлектрического материала и измеряется диэлектрической проницаемостью материала.

Разрешающая способность

Проницаемость указывается для любого конкретного материала как ОТНОСИТЕЛЬНАЯ ДОПУСТИМОСТЬ, которая является мерой эффективности диэлектрического материала. Это число без единиц измерения, которое показывает, насколько диэлектрическая проницаемость материала больше диэлектрической проницаемости воздуха (или вакуума), для которой задана диэлектрическая проницаемость 1 (единица). Например, если диэлектрический материал, такой как слюда, имеет относительную диэлектрическую проницаемость 6, это означает, что конденсатор будет иметь диэлектрическую проницаемость, а значит, и емкость, в шесть раз больше, чем у конденсатора с такими же размерами, но диэлектриком которого является воздух.

Диэлектрическая прочность

Другой важный аспект диэлектрика — ДИЭЛЕКТРИЧЕСКАЯ ПРОЧНОСТЬ. это указывает на способность диэлектрика выдерживать напряжение, приложенное к нему, когда конденсатор заряжен. В идеале диэлектрик должен быть как можно более тонким, чтобы обеспечить максимальную емкость для данного размера компонента. Однако чем тоньше диэлектрический слой, тем легче разрушаются его изоляционные свойства. Таким образом, диэлектрическая прочность определяет максимальное рабочее напряжение конденсатора.

Максимальное рабочее напряжение (VDCwkg max)

При использовании конденсаторов очень важно, чтобы максимальное рабочее напряжение, указанное производителем, не превышалось. В противном случае существует большая опасность внезапного пробоя изоляции внутри конденсатора. Поскольку в это время на конденсаторе существует максимальное напряжение (отсюда и пробой), большие токи будут протекать с реальным риском возгорания или взрыва в некоторых цепях.

Заряд конденсатора.

Заряд (Q) конденсатора зависит от комбинации вышеперечисленных факторов, которые можно представить вместе как емкость (C) и приложенное напряжение (V). Для компонента данной емкости соотношение между напряжением и зарядом является постоянным. Увеличение приложенного напряжения приводит к пропорциональному увеличению заряда. Эту связь можно выразить формулой;

Q = CV

или

C = Q / V

или

V = Q / C

Где V — приложенное напряжение в вольтах.

C — емкость в Фарадах.

Q — количество заряда в кулонах.

Итак, любая из этих величин может быть найдена, если известны две другие. Формулы можно легко переставить, используя простой треугольник, аналогичный тому, который используется для расчета закона Ома при проведении расчетов резисторов.

.
Разное

Добавить комментарий

Ваш адрес email не будет опубликован. Обязательные поля помечены *